Область допустимых значений функции и область определения функции: область определения и область значений функций + ПРИМЕРЫ

Содержание

область определения и область значений функций + ПРИМЕРЫ

Функция-это модель. Определим X, как множество значений независимой переменной // независимая -значит любая.

Функция это правило, с помощью которого  по каждому значению независимой переменной из множества X можно найти единственное значение зависимой переменной. // т.е. для каждого х есть один у.

Из определения следует, что существует два понятия- независимая переменная (которую обозначаем х и она может принимать любые значения) и зависимая переменная (которую обозначаем y или f(х) и она высчитывается из функции, когда мы подставляем х).

НАПРИМЕР у=5+х 

1. Независимая -это х, значит берем любое значение, пусть х=3 

2. а теперь вычисляем у, значит у=5+х=5+3=8. (у зависима от х, потому что какой х подставим, такой у и получим)

Говорят, что переменная y функционально зависит от переменной x и обозначается это следующим образом: y = f (x).

НАПРИМЕР.

1.у=1/х.            (наз.

2.          (наз. парабола)

3.у=3х+7.         (наз. прямая)

4. у= √ х.           (наз. ветвь параболы)

Независимая переменная (кот. мы обозначаем х) имеет название аргумент функции. 

Область определения функции

Множество всех значений, которые принимает аргумент функции, называется областью определения функции и обозначается  D (f) или D (y).

 Рассмотрим D (у) для 1.,2.,3.,4.

1. D (у)= ( ∞; 0) и (0;+∞) //всё множество действительных чисел, кроме нуля.

2. D (у)= ( ∞; +∞)//всё мн-во действит.чисел

3. D (у)= ( ∞; +∞)//всё мн-во действит.чисел

4. D (у)= [0; +∞)// мн-во неотрицат.чисел

Зависимая переменная (кот. мы обозначаем у ) имеет название значение функции.

Область значения функции

Множество всех значений, которые может принять зависимая переменная, называется областью значения функции и обозначается E (f) или E (y).

Рассмотрим  Е (у) для 1.,2.,3.,4.

1. Е (у)= ( ∞; 0) и (0;+∞) //всё множество действительных чисел, кроме нуля.

2. Е (у)= [0; +∞)// мн-во неотрицат.чисел

3. Е (у)=( ∞; +∞)//всё мн-во действит.чисел 

4. Е (у)= [0; +∞)// мн-во неотрицат.чисел

Рассмотрим примеры подробнее

1) Постановка задачи. Найти функции у= 4х/(3+х)

Решение.

1. Найдем D (у)//т.е. какие значения может принимать х. для этого найдем ОДЗ(область допустимых значений дроби)

3+х≠0

х≠-3

значит D (у) данной функции  ( ∞; 3) и (3;+∞)// всё множество действительных чисел, кроме 3.

2. Найдем  Е (у)//т.е. какие значения может принимать у, при всех возможных х

решаем уравнение вида 4х/(3+х)=А, где  А є Е (у)

(3+х)А=4х

3А=4х-хА

3А=х(4-А)

х=3А/(4-А)

значит Е (у) данной функции ( ∞; 4) и (4;+∞)// всё множество действительных чисел, кроме 4.

2) Постановка задачи. Найти  D (у)и Е (у) функции, изображенной на графике

 

Область определения(значения х) смотрим по оси х- это промежуток [ 4; 7], 

Областью значения(значения у) смотрим по оси у- это промежуток [ 4; 4].

Нужна помощь в учебе?



Предыдущая тема: Графический способ решения уравнений: алгоритм и примеры графиков
Следующая тема:&nbsp&nbsp&nbspСвойства функции: разбираем на примере

Как найти Область Допустимых Значений (ОДЗ)

Допустимые и недопустимые значения переменных

В 7 классе заканчивается математика и начинается ее-величество-алгебра. Первым делом школьники изучают выражения с переменными.

Мы уже знаем, что математика состоит из выражений — буквенных и числовых. Каждому выражению, в котором есть переменная, соответствует область допустимых значений (ОДЗ). Если игнорировать ОДЗ, то в результате решения можно получить неверный ответ. Получается, чтобы быстро получить верный ответ, нужно всегда учитывать область допустимых значений. 

Чтобы дать верное определение области допустимых значений, разберемся, что  такое допустимые и недопустимые значения переменной. 

Рассмотрим все необходимые определения, связанные с допустимыми и недопустимыми значениями переменной.

Выражение с переменными — это буквенное выражение, в котором буквы обозначают величины, принимающие различные значения.

Значение числового выражения — это число, которое получается после выполнения всех действий в числовом выражении.

Выражение с переменными имеет смысл при данных значениях переменных, если при этих значениях переменных можно вычислить его значение.

Выражение с переменными не имеет смысла при данных значениях переменных, если при этих значениях переменных нельзя вычислить его значение.

Теперь, опираясь на данные определения, мы можем сформулировать, что такое допустимые и недопустимые значения переменной.

Допустимые значения переменных — это значения переменных, при которых выражение имеет смысл.

Если при переменных выражение не имеет смысла, то значения таких переменных называют недопустимыми

В выражении может быть больше одной переменной, поэтому допустимых и недопустимых значений может быть больше одного.  

Пример 1

Рассмотрим выражение

В выражении три переменные (a, b, c). 

Запишем значения переменных в виде: a = 1, b = 1, c = 2.

Такие значения переменных являются допустимыми, поскольку при подстановке этих значений в выражение, мы легко можем найти ответ:

Таким же образом можем выяснить, какие значения переменных  — недопустимые. 

a = 1, b = 2, c = 1.

Подставим значения переменных в выражение

На ноль делить нельзя. 

Что такое ОДЗ

ОДЗ — это невидимый инструмент при решении любого выражении с переменной. Чаще всего, ОДЗ не отображают графически, но всегда «держат в уме».


Область допустимых значений (ОДЗ) — это множество всех допустимых значений переменных для данного выражения.

Запоминаем!

ОДЗ относится к выражениям. Область определения функции относится к функциям и не относится к выражениям.

Пример 2

Рассмотрим выражение

ОДЗ такого выражения выглядит следующим образом: ( — ∞; 3) ∪ (3; +∞).

Читать запись нужно вот так:
Область допустимых значений переменной x для выражения  — это числовое множество ( — ∞; 3) ∪ (3; +∞).

Пример 3


Рассмотрим выражение

ОДЗ такого выражения будет выглядеть вот так: b ≠ c; a — любое число.

Такая запись означает, что область допустимых значений переменных b, c и a = это все значения переменных, при которых соблюдаются условия b ≠ c; a — любое число.


Каждый новый год в школе прибавляет ученикам забот — задачки становятся сложнее, формулы длиннее, а правила — скучнее. В детской онлайн-школе Skysmart ученики занимаются на красочной интерактивной платформе, пользуются электронным учебником и чертят на настоящей онлайн-доске. Такая алгебра не может не понравиться.

Записывайтесь на бесплатный вводный урок и пробуйте новый формат обучения.

Как найти ОДЗ: примеры решения

Найти ОДЗ — это значит, что нужно указать все допустимые значения переменных для выражения. Часто, чтобы найти ОДЗ, нужно выполнить преобразование выражения.

Чтобы быстро и верно определять ОДЗ, запомните условия, при которых значение выражения не может быть найдено. 

Мы не можем вычислить значение выражения, если:

  • требуется извлечение квадратного корня из отрицательного числа
  • присутствует деление на ноль (математическое правило номер раз: никогда не делите на ноль)
  • отрицательный целый показатель в степени при отрицательном числе
  • требуется вычисление логарифма отрицательного числа
  • область определения тангенса = π * k, где k ∈ z
  • область определения котангенса π * k, где k ∈ z
  • нахождение арксинуса и арккосинуса числа, выходящего за пределы числового промежутка [- 1, 1].

Теперь, приступая к поиску ОДЗ, вы можете сверять выражение по всем этим пунктам. 

Давайте потренируемся находить ОДЗ.

Пример 4

Найдем область допустимых значений переменной выражения a3 + 4 * a * b − 6.

Как решаем:

В куб возводится любое число. Ограничений при вычитании и сложении нет. Это значит, что мы можем вычислить значение выражения a3 + 4 * a * b − 6 при любых значениях переменной. 

ОДЗ переменных  a и b — это множество таких пар допустимых значений (a, b), где a — любое число и  b — любое число. 

Ответ: (a и b), где a — любое число и b — любое число.

Пример 5

Найдем область допустимых значений (ОДЗ) переменной выражения 

Здесь нужно обратить внимание на наличие нуля в знаменатели дроби. Одним из условий, при котором вычисление значения выражения невозможно явлется наличие деления на ноль. 

Это значит, что мы может сказать, что ОДЗ переменной a в выражении — пустое множество.

Пустое множество изображается в виде вот такого символа Ø.

Пример 6

Найдем область допустимых значений (ОДЗ) переменных в выражении

Если  есть квадратный корень, то нам нужно следить за тем, чтобы под знаком корня не было отрицательного числа. Это значит, что при подстановке значений a и b должны быть условия, при которых a + 3 * b + 5 ≥ 0.

Ответ: ОДЗ переменных a и b — это множество всех пар, при которых a + 3 * b + 5 ≥ 0.

Лайфхак

Чтобы не потратить зря время на решение нерешаемого примера, всегда обращайтесь к списку условий, при которых выражение не может быть решено.

Пример 7

Найдем ОДЗ переменной a в выражении

Прежде всего, нам нужно подобрать такое условие, при котором в знаменателе дроби не  будет ноля —

Мы знаем, что выражение под знаком корня должно быть положительным. Это дает нам второе условие: a + 1 ≥ 0.

Мы не можем вычислить логарифм отрицательного выражения. Получаем третье условие: a2

+ 2 > 0.

Выражении в основании логарифма не должно быть отрицательным и не должно равняться единице. Получаем условие 4: a + 6 > 0.

Условие 5: a + 6 ≠ 1.

Определим ОДЗ, опираясь на все означенные условия:
a +1 — 1 0.


Ответ: ОДЗ: [ — 1; 0) ∪ (0; +∞)

Как видите, записывая ОДЗ, мы ставим квадратные и круглые скобки.

Запомните

  • Если число входит в ОДЗ, то около числа ставим квадратные скобки.
  • Если число не входит в ОДЗ, то около него ставятся круглые скобки. 

Например, если х > 6, но х < 8, то  записываем интервал [6; 8).

Зачем учитывать ОДЗ при преобразовании выражения

Иногда выражение просто невозможно решить, если не выполнить ряд тождественных преобразований. К ним относятся: перестановки, раскрытие скобок, группировка, вынесение общего множителя за скобки, приведение подобных слагаемых.

Кроме того, что видов таких преобразований довольно много: нужно понимать, в каких случаях какое преобразование возможно. В этом может помочь определение ОДЗ.

Тождественное преобразование может:

  • расширить ОДЗ
  • никак не повлиять на ОДЗ
  • сузить ОДЗ

Рассмотрим каждый случай в отдельности.

Пример 8

Рассмотрим выражение a + 4/a — 4/a

Поскольку мы должны следить за тем, чтобы в выражении не возникало деление ноль, определяем условие a ≠ 0.

Это условие отвечает множеству (−∞ ; 0) ∪ (0 ; +∞).

В выражении есть подобные слагаемые, если привести подобные слагаемые, то мы получаем выражение вида a. 

ОДЗ для a — это R — множество всех вещественных чисел. 

Преобразование расширило ОДЗ — добавился ноль. 

Пример 9

Рассмотрим выражение a2 + a + 4 * a

ОДЗ a для этого выражения — множество R.

В выражении есть подобные слагаемые, выполним тождественное преобразование. 

После приведения подобных слагаемых выражение приняло вид  a2 + 5 * a 

ОДЗ переменной a для этого выражения — множество R.

Это значит, что тождественное преобразование никак не повлияло на ОДЗ. 

Пример 10

Рассмотрим выражение

ОДЗ a определяется неравенством (a — 1) * (a — 4) ≥ 0.

Решить такое неравенство можно методом интервалов, что дает нам ОДЗ (−∞; 1] ∪ [4 ; +∞).

Затем выполним преобразование исходного выражения по свойству корней: корень произведения = произведению корней.

Приведем выражение к виду

ОДЗ переменной a для этого выражения определяется неравенствами:
a — 1 ≥ 0
a — 4 ≥ 0

Решив систему линейных неравенств, получаем множество [4; + ∞).

Отсюда видно, что тождественные преобразования сузили ОДЗ.
От (−∞; 1] ∪ [4 ; +∞) до [4; + ∞).

Решив преобразовать выражение, внимательно следите за тем, чтобы не допустить сужение ОДЗ.

Запомните, что выполняя преобразование, следует выбирать такие, которые не изменят ОДЗ.

Тебе следует повторить тему — формулы сокращенного умножения!

В детской школе Skysmart опытные преподаватели научат ребенка решать любые, даже самые сложные, задачки, справляться с формулами и теоремами. На уроках нет скучной зубрежки и непонятных правил — только эффективная подготовка к тестам, контрольными и экзаменам.

Заполнить пробелы в обучении и продвинуться вперед по программе легко и весело со Skysmart. Записывайтесь на бесплатный вводный урок и начните заниматься уже завтра!

Область допустимых значений (ОДЗ): теория, примеры, решения

Любое выражение с переменной имеет свою область допустимых значений, где оно существует. ОДЗ необходимо всегда учитывать при решении. При его отсутствии можно получить неверный результат.

В данной статье будет показано, как правильно находить ОДЗ, использовать на примерах. Также будет рассмотрена важность указания ОДЗ при решении.

Допустимые и недопустимые значения переменных

Данное определение связано с допустимыми значениями переменной. При введении определения посмотрим, к какому результату приведет.

Начиная с 7 класса, мы начинаем работать с числами и числовыми выражениями. Начальные определения с переменными переходят к значению выражений с выбранными переменными.

Когда имеются выражения с выбранными переменными, то некоторые из них могут не удовлетворять. Например, выражение вида 1:а, если а=0, тогда оно не имеет смысла, так как делить на ноль нельзя. То есть выражение должно иметь такие значения, которые подойдут в любом случае и дадут ответ. Иначе говоря, имеют смысл с имеющимися переменными.

Определение 1

Если имеется выражение с переменными, то оно имеет смысл только тогда, когда при их подстановке значение может быть вычислено.

Определение 2

Если имеется выражение с переменными, то оно не имеет смысл, когда при их подстановке значение не может быть вычислено.

То есть отсюда следует полное определение

Определение 3

Существующими допустимыми переменными называют такие значения, при которых выражение имеет смысл. А если смысла не имеет, значит они считаются недопустимыми.

Для уточнения вышесказанного: если переменных более одной, тогда может быть и пара подходящих значений.

Пример 1

Для примера рассмотрим выражение вида 1x-y+z, где имеются три переменные. Иначе можно записать, как x=0, y=1, z=2, другая же запись имеет вид (0,1,2). Данные значения называют допустимыми, значит, можно найти значение выражения. Получим, что 10-1+2=11=1. Отсюда видим, что (1,1,2) недопустимы. Подстановка дает  в результате деление на ноль, то есть 11-2+1=10. 

Что такое ОДЗ?

Область допустимых значений – важный элемент при вычислении алгебраических выражений. Поэтому стоит обратить на это внимание при расчетах.

Определение 4

Область ОДЗ – это множество значений, допустимых для данного выражения.

Рассмотрим на примере выражения. 

Пример 2

Если имеем выражение вида 5z-3, тогда ОДЗ имеет вид (−∞, 3)∪(3, +∞). Эта область допустимых значений, удовлетворяющая переменной z для заданного выражения.

Если имеется выражения вида zx-y, тогда видно, что x≠y, z принимает любое значение. Это и называют ОДЗ выражения. Его необходимо учитывать, чтобы не получить  при подстановке деление на ноль.

Область допустимых значений и область определения имеет один и тот же смысл. Только второй из них используется для выражений, а первый – для уравнений или неравенств. При помощи ОДЗ выражение или неравенство имеет смысл. Область определения функции совпадает  с областью допустимых значений переменной х к выражению f(x).

Как найти ОДЗ? Примеры, решения

Найти ОДЗ означает найти все допустимые значения, подходящие для заданной функции или неравенства. При невыполнении этих условий можно получить неверный результат. Для нахождения ОДЗ зачастую необходимо пройти через преобразования в заданном выражении.

Существуют выражения, где их вычисление невозможно:

  • если имеется деление на ноль;
  • извлечение корня из отрицательного числа;
  • наличие отрицательного целого показателя – только для положительных чисел;
  • вычисление логарифма отрицательного числа;
  • область определения тангенса π2+π·k, k∈Z и котангенса π·k, k∈Z;
  • нахождение значения арксинуса и арккосинуса числа при значении, не принадлежащем [-1; 1].

Все это говорит о том, как важно наличие ОДЗ.

Пример 3

Найти ОДЗ выражения x3+2·x·y−4.

Решение

В куб можно возводить любое число. Данное выражение не имеет дроби, поэтому значения x и у могут быть любыми. То есть ОДЗ – это любое число.

Ответ: x и y – любые значения.

Пример 4

Найти ОДЗ выражения 13-x+10.

Решение

Видно, что имеется одна дробь, где в знаменателе ноль. Это говорит о том, что  при любом значении х мы получим деление на ноль. Значит, можно сделать вывод о том, что это выражение считается неопределенным, то есть не имеет ОДЗ.

Ответ: ∅.

Нужна помощь преподавателя?

Опиши задание — и наши эксперты тебе помогут!

Описать задание   Пример 5

Найти ОДЗ заданного выражения x+2·y+3-5·x.

Решение

Наличие квадратного корня говорит о том, что это выражение обязательно должно быть больше или равно нулю. При отрицательном значении оно не имеет смысла. Значит, необходимо записать неравенство вида x+2·y+3≥0. То есть это и есть искомая область допустимых значений.

Ответ: множество x и y, где x+2·y+3≥0.

Пример 6

Определить ОДЗ выражения вида 1x+1-1+logx+8(x2+3).

Решение

По условию имеем дробь, поэтому ее знаменатель не должен равняться нулю. Получаем, что x+1-1≠0 . Подкоренное выражение всегда имеет смысл, когда больше или равно нулю, то есть x+1≥0. Так как имеет логарифм, то его выражение должно быть строго положительным, то есть x2+3>0. Основание логарифма также должно иметь положительное значение и отличное от 1, тогда добавляем еще условия x+8>0 и x+8≠1.  Отсюда следует, что искомое ОДЗ примет вид:

x+1-1≠0,x+1≥0,x2+3>0,x+8>0,x+8≠1

Иначе говоря, называют системой неравенств с одной переменной. Решение приведет к такой записи ОДЗ [−1, 0)∪(0, +∞).

Ответ: [−1, 0)∪(0, +∞)

Почему важно учитывать ОДЗ при проведении преобразований?

При тождественных преобразованиях важно находить ОДЗ. Бывают случаи, когда существование ОДЗ не имеет место. Чтобы понять, имеет ли решение заданное выражение, нужно сравнить ОДЗ переменных исходного выражения и ОДЗ полученного.

Тождественные преобразования:

  • могут не влиять на ОДЗ;
  • могут привести в расширению или дополнению ОДЗ;
  • могут сузить ОДЗ.

Рассмотрим на примере.

Пример 7

Если имеем выражение вида x2+x+3·x, тогда его ОДЗ определено на всей области определения. Даже при приведении подобных слагаемых и упрощении выражения ОДЗ не меняется.

Пример 8

Если взять пример выражения x+3x−3x, то дела обстоят иначе. У нас имеется дробное выражение. А мы знаем, что деление на ноль недопустимо. Тогда ОДЗ имеет вид (−∞, 0)∪(0, +∞). Видно, что ноль не является решением, поэтому добавляем его с круглой скобкой.

Рассмотрим пример с наличием подкоренного выражения. 

Пример 9

Если имеется x-1·x-3, тогда следует обратить внимание на ОДЗ, так как его необходимо записать в виде неравенства (x−1)·(x−3)≥0.   Возможно решение методом интервалов, тогда получаем, что ОДЗ примет вид (−∞, 1]∪[3, +∞). После преобразования x-1·x-3 и применения свойства корней имеем, что ОДЗ можно дополнить и записать все в виде системы неравенства вида x-1≥0,x-3≥0. При ее решении получаем, что [3, +∞). Значит, ОДЗ полностью записывается так: (−∞, 1]∪[3, +∞).

Нужно избегать преобразований, которые сужают ОДЗ.

Пример 10

Рассмотрим пример выражения x-1·x-3, когда х=-1. При подстановке получим, что -1-1·-1-3=8=22. Если это выражение преобразовать и привести к виду x-1·x-3, тогда при вычислении получим, что 2-1·2-3 выражение смысла не имеет, так как подкоренное выражение не должно быть отрицательным.

Следует придерживаться тождественных преобразований, которые ОДЗ не изменят.

Если имеются примеры, которые его расширяют, тогда его нужно добавлять в ОДЗ.

Пример 11

Рассмотрим на примере дроби вида xx3+x. Если сократить на x, тогда получаем, что 1×2+1. Тогда ОДЗ расширяется и становится (−∞ 0)∪(0, +∞). 2$

Функции вида $f(x)=\sin x, g(x)=\cos x, h(x)=\tan x, k(x)=\cot x$ называются тригонометрическими функциями. Область определения $f(x)=\sin x $ и $g(x)=\cos x$ это все действительные числа $\mathbb{R}$. А области определения $h(x)=\tan x $ и $k(x)=\cot x$ следующие: $h(x)=\tan x=\dfrac{\sin x}{\cos x}, \cos x=0 \rightarrow x=k\pi+\dfrac{\pi}{2} \rightarrow$

$D_h=\mathbb{R}-\lbrace x|x=k\pi+\dfrac{\pi}{2}, k \in \mathbb{Z} \rbrace$

$h(x)=\cot x=\dfrac{\cos x}{\sin x}, \sin x=0 \rightarrow x=k\pi \rightarrow$

$D_k=\mathbb{R}-\lbrace x|x=k\pi, k \in \mathbb{Z} \rbrace$

Также отметим, что $-1 \leq \sin x \leq 1 $ и $ -1 \leq \cos x \leq 1$. Следовательно,

$R_f=[-1,1] \,\,\,\,\,\, R_g=[-1,1]$

Множество значений of $h(x)=\tan x $ и $k(x)=\cot x$ это все действительные числа $\mathbb{R}$.
Пример:
Найти область определения и множество значений $f(x)=\sin x+\cos x$.

Решение:
Область определения $\sin x $ и $\cos x$ это все действительные числа, следовательно область определения

$f(x)=\sin x+\cos x$

также все действительные числа. 4 \pi x = 0 \rightarrow \sin \pi x=0 \rightarrow \pi x=k \pi \rightarrow x=k \in \mathbb{Z}$

Значит

$D_f=\mathbb{Z}$

Согласно $D_f=\mathbb{Z}$, можно переписать функцию как

$f(x)=\cos \pi x=\pm 1$

Теперь очевидно, что

$R_f= \lbrace \pm 1 \rbrace$


Пример:
Найти область определения и множество значений $f(x)=\sin (\log (\log x))$.

Решение:
Согласно тому, что уже было сказано относительно логарифмической функции

$D_f= \lbrace x| x \in \mathbb{R}; \log x>0,x>0 \rbrace$

$= \lbrace x| x\in \mathbb{R}, x>1,x>0 \rbrace =(1,+\infty)$

Также стоит отметить, что

$|\sin (\log (\log x))| \leq 1 \rightarrow |y| \leq 1 \rightarrow -1 \leq y \leq 1$

Значит

$R_f=[-1,1]$

График $f$ это
Определение:
Пусть $f$ функция, у которой область определения это $D_f$. Функция $f$ является инъективной тогда и только тогда, если для всех $x_1$ и $x_2$ в $D_f$, если $f(x_1)=f(x_2)$, то $x_1=x_2$. {\log x} \rightarrow y=(f \circ g)_{(x)}\,\,\, x \in (0,1) \rightarrow 0 Теперь, для того, чтобы найти множество значений $g \circ f$, отметим, что

$Z=(g\circ f)_{(x)}=x \rightarrow x=Z\in (1,+\infty) \rightarrow Z>1 \rightarrow R_{g \circ f}=(1,+\infty)$

Графиком $f$ является
Графиком $g$ является
График $f \circ g$ это
График $g \circ f$ это

Пример:
Если $f(x)=x-1$ and $(f \circ g)_{(x)}=\dfrac{1}{x-1}$, то найти область определения и множество значений $g \circ f$.
Решение:
Сначала найдем $ g \circ f$

$f(x)=x-1 \rightarrow f(g(x))=g(x)-1 \rightarrow (f \circ g)_{(x)}=g(x)-1 \rightarrow \\ \dfrac{1}{x-1}=g(x)-1 \rightarrow g(x)=\dfrac{x}{x+1}$

Значит

$y=(g \circ f)_{(x)}=g(f(x))=\dfrac{f(x)}{f(x)-1}=\dfrac{x-1}{x+1}$

Следовательно

$D_{g \circ f}=\lbrace x|x \in \mathbb{R}, x \neq 2 \rbrace \rightarrow D_{g \circ f}=\mathbb{R}-\lbrace 2 \rbrace$

Также

$y=\dfrac{x-1}{x-2} \rightarrow x=\dfrac{2y-1}{y-1}$

$R_{g \circ f}=\lbrace y | y \in \mathbb{R}, y \neq 1 \rbrace \rightarrow$

$R_{g \circ f}=\mathbb{R}-\lbrace 1 \rbrace$

График $f$ это
График $f \circ g$ это
Графиком $g$ является
Графиком $g \circ f$ является

Упражнения

1) Если $f(x)=2^{\log_2 x}$ and $g(x)=\dfrac{x-1}{x^2-x}$, то найти область определения и множество значений $f \circ g$. 2 2kx \,\,\, -1 \leq \sin 2kx \leq 1$

$\rightarrow \sin 2kx= \pm 1 \rightarrow y=\dfrac{1}{4} , \sin 2x=0 \rightarrow y=1$

$\rightarrow \dfrac{1}{4} \leq y \leq 1 \rightarrow R_f=[\dfrac{1}{4},1]$

Part 1

Как найти область допустимых значений функции

Область допустимых значений алгебраического выражения (сокращенно ОДЗ) – это множество значений переменной, при которых это выражение определено.

В школьном курсе алгебры есть всего пять элементарных функций, которые имеют ограниченную область определения. Вот они:

1. ОДЗ:

Выражение, стоящее под знаком корня четной кратности, должно быть больше или равно нулю.

2. ОДЗ:

Выражение, стоящее в знаменателе дроби, не может быть равно нулю.

3. ОДЗ:

Выражение, стоящее под знаком логарифма, должно быть строго больше нуля; выражение, стоящее в основании логарифма должно быть строго больше нуля и отлично от единицы.

4. , ОДЗ:

5. Есть две функции, которые содержат «скрытую» дробь:

и

6. ОДЗ:

Степень корня – натуральное число, отличное от 1.

Таким образом, функции и имеют разную область определения.

Если выражение содержит одну или несколько функций, которые определены на ограниченном множестве значений аргумента, то для того, чтобы найти ОДЗ выражения, нужно учесть все ограничения, которые накладываются этими функциями.

Чтобы найти область допустимых значений выражения, нужно исследовать, присутствуют ли в выражении функции, которые я перечислила выше. И по мере обнаружения этих функций, записывать задаваемые ими ограничения, двигаясь «снаружи» «внутрь».

Поясню на примере:

Найти область определения функции:

Чтобы найти область определения функции, нужно найти область допустимых значений выражения, которое стоит в правой части уравнения функции

Я специально выбрала «страшную», на первый взгляд, функцию, чтобы показать вам, на какие простые операции разбивается процесс нахождения области допустимых значений.

«Просканируем» выражение, стоящее в правой части равенства:

1. Мы видим дробь:

Знаменатель дроби не равен нулю. Записываем:

2. Мы видим в знаменателе логарифм:


Выражение, стоящее под знаком логарифма должно быть строго больше нуля; выражение, стоящее в основании логарифма должно быть строго больше нуля и отлично от единицы.

3.Мы видим квадратный корень:

Выражение, стоящее под знаком корня четной кратности, должно быть больше или равно нулю.

Теперь запишем все ограничения в систему неравенств:

Решение этой системы неравенств посмотрите в ВИДЕУРОКЕ:

  • Каждому выражению с переменными соответствует область допустимых значений (ОДЗ) переменных, которую ОБЯЗАТЕЛЬНО нужно учитывать при работе с этим выражением. Акцент на слове «обязательно» сделан не случайно: при решении примеров и задач халатное отношение к ОДЗ может привести к получению неверных результатов.

    Чтобы у нас не возникало подобных проблем, давайте внимательно изучим все, что связано с ОДЗ. Для начала узнаем, что это такое, после этого разберем на характерных примерах, как найти ОДЗ переменных для заданного выражения, а в заключение остановимся на важности учета ОДЗ при преобразовании выражений.

    Навигация по странице.

    Допустимые и недопустимые значения переменных

    Определение области допустимых значений переменных для выражения дается через термин допустимые значения переменной. Введем это вспомогательное определение, для чего проследим, что нас приводит к нему.

    На уроках математики в школе вплоть до 7 класса познаются азы работы преимущественно с числами и числовыми выражениями. А с 7 класса начинается изучение такой математической дисциплины как алгебра, и начинается оно с того, что вводится определение выражения с переменными, а также связанное с ним определение значения выражения при выбранных значениях переменных.

    Последнее определение нуждается в уточнении следующего плана. Существуют выражения, значения которых при некоторых выбранных значениях переменных вычислить невозможно. Например, невозможно вычислить значение выражения 1:a при a=0 , так как делить на нуль нельзя. Это послужило причиной введения в обиход терминов «выражение, имеющее смысл при данных значениях переменных» и «выражение, не имеющее смысла при данных значениях переменных». Говорят, что

    выражение с переменными имеет смысл при данных значениях переменных, если при этих значениях переменных можно вычислить его значение

    выражение с переменными не имеет смысла при данных значениях переменных, если при этих значениях переменных нельзя вычислить его значение.

    Вот теперь мы обладаем всеми сведениями, позволяющими дать определение допустимых и недопустимых значений переменных:

    Допустимые значения переменных – это такие значения переменных, при которых выражение имеет смысл. А значения переменных, при которых выражение не имеет смысла, называют недопустимыми значениями переменных.

    Здесь лишь стоит уточнить, что если выражение содержит две, три, и большее число переменных, то речь идет о парах, тройках и т.д. допустимых значений переменных. Приведем пример. Рассмотрим выражение с тремя переменными x , y и z . Тройка значений переменных x=0 , y=1 , z=2 , она же в другой записи (0, 1, 2) , является допустимой, так как при данных значениях переменных мы можем найти значение выражения: . А тройка (1, 2, 1) – недопустимая, так как при подстановке этих значений в выражение мы придем к делению на нуль: .

    Определения, озвученные в этом пункте, полностью согласуются с информацией из учебников [1, с. 6; 2, с. 11-12; 3, c. 4] .

    Что такое ОДЗ?

    Практически у всех, так или иначе имеющих отношение к алгебре, на слуху словосочетание «область допустимых значений», также довольно часто аббревиатуру ОДЗ можно встретить в описаниях решений, но как такового определения области допустимых значений (ОДЗ) нет в основных учебниках, используемых в школе. Поэтому интересно, откуда берет начало этот термин. Ну а с позиций практики интереснее знать, какой смысл в него вкладывают.

    Под областью допустимых значений (ОДЗ) понимают множество всех допустимых значений переменных для данного выражения.

    Приведем пример. Допустим, дано выражение , и записано ОДЗ: (−∞, 3)∪(3, +∞) . Последнюю запись стоит понимать так: область допустимых значений переменной z для выражения есть числовое множество (−∞, 3)∪(3, +∞) .

    Другой пример. Рассмотрим выражение и относящуюся к нему запись ОДЗ: x≠y , z – любое. Она означает, что ОДЗ переменных x , y и z для данного выражения – это все такие тройки значений переменных x , y и z , для которых выполняются указанные условия x≠y , z – любое.

    Завершить этот пункт хочется разговором про область допустимых значений и область определения. Часто между этими терминами стирают различия. Например, говорят про область определения выражения [4, с. 87] , под которой фактически понимают ОДЗ переменных этого выражения. Также можно столкнуться с областью определения уравнения или неравенства [5, с. 204, 220; 6, с. 188, 190] , под ней подразумевают ОДЗ переменных, на которой одновременно имеют смысл обе части уравнения или неравенства. Как тут не спутать одно с другим? Давайте будем придерживаться следующего подхода: к функциям относить область определения функции, а к выражениям – ОДЗ переменных. И на загладку приведем такое утверждение: область определения функции y=f(x) совпадает с областью допустимых значений переменной x для выражения f(x) .

    Как найти ОДЗ? Примеры, решения

    Прежде чем обратиться к главной теме этого пункта, нужно понимать, что значит найти ОДЗ, хотя это достаточно отчетливо ясно из определения. Это значит, что надо указать множество всех допустимых значений переменных для заданного выражения. На это можно посмотреть и с другой стороны: найти ОДЗ – это значит указать условия, которые исключают те и только те значения переменных, при которых выражение не имеет смысла. Теперь можно двигаться дальше.

    Заданий с формулировкой «найти ОДЗ» не так много. Однако почти постоянно приходится преобразовывать выражения, а это неявно требует нахождения области допустимых значений для ее контроля. В этом свете вопрос, как найти ОДЗ, очень злободневен.

    В поисках ответа на него поразмыслим, значения каких выражений мы не можем вычислить.

  • Во-первых, мы не можем вычислить значение выражения, в котором присутствует деление на нуль (или дробь со знаменателем нуль, что по сути то же самое), так как этому действию мы не придали смысла.
  • Во-вторых, мы не можем извлечь квадратный корень из отрицательного числа, как и корень другой четной степени, о чем мы говорим когда вводили корень из числа. Здесь же заметим, что показателями корня могут быть лишь числа 2 , 3 , 4 , и так далее, значит, значения выражений с корнями, имеющими другие показатели, мы тоже не можем вычислить.
  • В-третьих, вспомним про степень числа. Если степень числа с положительным целым показателем мы определили для любого действительного числа, то степень с целым отрицательным показателем мы определили уже с ограничением: для любого действительного числа, кроме числа нуль. Степени с положительным нецелым показателем мы придали смысл лишь для неотрицательных чисел, а с отрицательным нецелым показателем – лишь для положительных чисел. А еще мы не можем вычислить нуль в степени нуль.
  • В-четвертых, обратим внимание на логарифм числа. Его мы определили так, что не придали смысла логарифму отрицательного числа и числа нуль по любому основанию, а также логарифму положительного числа по отрицательному основанию и по основанию 1 .
  • В-пятых, мы не определили тангенс чисел , а также котангенс чисел (см. статью значения тригонометрических функций).
  • В-шестых, мы не можем найти значение арксинуса и арккосинуса числа, выходящего за рамки числового промежутка [−1, 1] в силу того, что мы так определили arcsin и arccos (см. статью arcsin, arccos, arctg, arcctg: определения, примеры).
  • Что нам это дает? А то, что перечисленные выше моменты и нужно учитывать при поиске ОДЗ. Как это делать, станет понятно из следующих примеров.

    Укажите все допустимые значения переменных для выражения x 3 +2·x·y−4 .

    Возвести в куб мы можем любое число, также мы умеем умножать любые числа, как и складывать и вычитать. Поэтому, мы можем вычислить значение заданного выражения при любых значениях переменных x и y . А это значит, что выражение x 3 +2·x·y−4 имеет смысл при любых значениях входящих в него переменных. Поэтому, ОДЗ переменных x и y для этого выражения – это множество всех таких пар (x, y) , где x – любое число и y – любое число.

    (x, y) , где x – любое, y – любое.

    Найти ОДЗ переменной x для выражения .

    Мы видим, что данное выражение содержит дробь с нулем в знаменателе. А это значит, что ни при каком значении переменной x мы не сможем вычислить значение этого выражения, так как оно будет содержать деление на нуль. Вывод: это выражение не определено ни при каких значениях переменной x . Другими словами, ОДЗ переменной x для этого выражения есть пустое множество.

    Найти ОДЗ .

    Здесь нас настораживает присутствие квадратного корня. Чтобы избежать появления под корнем отрицательного числа, надо для переменных x и y потребовать выполнение условия x+2·y+3≥0 . Оно и задает искомую область допустимых значений.

    множество всех пар (x, y) , для которых x+2·y+3≥0 .

    В более сложных случаях приходится учитывать одновременно несколько условий из приведенного выше списка. Это дает системы неравенств, задающие ОДЗ.

    Определите ОДЗ переменной x для выражения .

    Во-первых, выражение в знаменателе дроби не должно обращаться в нуль, это дает первое условие . Во-вторых, выражение под знаком квадратного корня должно быть неотрицательным: x+1≥0 . В-третьих, выражение под знаком логарифма должно быть положительным, это дает третье условие x 2 +3>0 . Наконец, выражение в основании логарифма должно быть положительным и отличным от единицы, так вырисовываются еще два условия x+8>0 и x+8≠1 . Таким образом, искомая ОДЗ определяется системой следующего вида . Это система неравенств с одной переменной, решив ее, записываем ОДЗ: [−1, 0)∪(0, +∞) .

    Здесь лишь заметим, что во многих случаях на практике нет необходимости в решении составленных систем.

    В заключении остается сказать, что такой подход используется и тогда, когда нужно найти область определения функции.

    Почему важно учитывать ОДЗ при проведении преобразований?

    Решая различные задачи, нам очень часто приходится проводить тождественные преобразования выражений. Но бывает, что какое-то преобразование в одних случаях допустимо, а в других – нет. Существенную помощь в плане контроля допустимости проводимых преобразований оказывает ОДЗ. Остановимся на этом подробнее.

    Суть подхода состоит в следующем: сравниваются ОДЗ переменных для исходного выражения с ОДЗ переменных для выражения, полученного в результате выполнения тождественных преобразований, и на основании результатов сравнения делаются соответствующие выводы.

    Вообще, тождественные преобразования могут

    • не влиять на ОДЗ;
    • приводить к расширению ОДЗ;
    • приводить к сужению ОДЗ.

    Давайте поясним каждый случай примером.

    Рассмотрим выражение x 2 +x+3·x , ОДЗ переменной x для этого выражения есть множество R . Теперь проделаем с этим выражением следующее тождественное преобразование – приведем подобные слагаемые, в результате оно примет вид x 2 +4·x . Очевидно, ОДЗ переменной x этого выражения тоже является множество R . Таким образом, проведенное преобразование не изменило ОДЗ.

    Переходим дальше. Возьмем выражение x+3/x−3/x . В этом случае ОДЗ определяется условием x≠0 , которое отвечает множеству (−∞, 0)∪(0, +∞) . Это выражение тоже содержит подобные слагаемые, после приведения которых приходим к выражению x , для которого ОДЗ есть R . Что мы видим: в результате проведенного преобразования произошло расширение ОДЗ (к ОДЗ переменной x для исходного выражения добавилось число нуль).

    Осталось рассмотреть пример сужения области допустимых значений после проведения преобразований. Возьмем выражение . ОДЗ переменной x определяется неравенством (x−1)·(x−3)≥0 , для его решения подходит, например, метод интервалов, в результате имеем (−∞, 1]∪[3, +∞) . А теперь преобразуем исходное выражение к виду , воспользовавшись одним из свойств корней: корень произведения равен произведению корней. ОДЗ переменной x для этого выражения определяет система линейных неравенств , решение которой дает множество [3, +∞) . Таким образом, в результате проведенного преобразования произошло сужение ОДЗ с множества (−∞, 1]∪[3, +∞) до множества [3, +∞) .

    При преобразовании выражений надо строго избегать преобразований, сужающих ОДЗ. Почему? Для пояснения приведем пример.

    Допустим нам нужно вычислить значение выражения при x=−1 . Если сразу подставить вместо переменной x число −1 , то мы найдем значение . А теперь представим, что мы из каких-то соображений предварительно преобразовали исходное выражение к виду , сузив тем самым ОДЗ. Вычисляем его значение, для этого подставляем вместо переменной x число −1 , и получаем выражение , которое не имеет смысла, так как под знаком корня оказывается отрицательное число. Такой подход привел нас к проблеме, которая возникла из-за того, что 2 входит в ОДЗ переменной x для исходного выражения, но уже не попадает в «суженную» ОДЗ переменной x для выражения, полученного после преобразования.

    Так что надо придерживаться таких тождественных преобразований выражения, которые не изменяют ОДЗ.

    А как быть с преобразованиями выражений, при которых расширяется ОДЗ? Их можно проводить, но при этом стоит придерживаться такого взгляда: полученное в результате преобразования выражение рассматривать на ОДЗ переменных исходного выражения.

    Например, сокращение алгебраической дроби на x дает дробь и приводит к расширению ОДЗ от множества (−∞ 0)∪(0, +∞) до множества R . При этом можно продолжать работать с полученной дробью , но на ОДЗ переменной x для исходного выражения, то есть, на множестве (−∞ 0)∪(0, +∞) .

    Еще пример. При замене суммы логарифмов lnx+ln(x+3) логарифмом произведения ln(x·(x+3)) (см. свойства логарифмов) происходит расширение ОДЗ с (0, +∞) до (−∞, −3)∪(0, +∞) . Поэтому с полученным выражением ln(x·(x+3)) дальше стоит работать на ОДЗ переменной x исходного выражения, то есть, на множестве (0, +∞) .

    Итак, на каждом шаге преобразования выражения постоянно спрашивайте себя: «Не изменяет ли это преобразование ОДЗ»? Если не изменяет, то выполняйте его. Если сужает, то откажитесь от него. А если расширяет, то выполняйте его, но оставайтесь в рамках ОДЗ переменных для исходного выражения.

    Любое выражение с переменной имеет свою область допустимых значений, где оно существует. ОДЗ необходимо всегда учитывать при решении. При его отсутствии можно получить неверный результат.

    В данной статье будет показано, как правильно находить ОДЗ, использовать на примерах. Также будет рассмотрена важность указания ОДЗ при решении.

    Допустимые и недопустимые значения переменных

    Данное определение связано с допустимыми значениями переменной. При введении определения посмотрим, к какому результату приведет.

    Начиная с 7 класса, мы начинаем работать с числами и числовыми выражениями. Начальные определения с переменными переходят к значению выражений с выбранными переменными.

    Когда имеются выражения с выбранными переменными, то некоторые из них могут не удовлетворять. Например, выражение вида 1 : а , если а = 0 , тогда оно не имеет смысла, так как делить на ноль нельзя. То есть выражение должно иметь такие значения, которые подойдут в любом случае и дадут ответ. Иначе говоря, имеют смысл с имеющимися переменными.

    Если имеется выражение с переменными, то оно имеет смысл только тогда, когда при их подстановке значение может быть вычислено.

    Если имеется выражение с переменными, то оно не имеет смысл, когда при их подстановке значение не может быть вычислено.

    То есть отсюда следует полное определение

    Существующими допустимыми переменными называют такие значения, при которых выражение имеет смысл. А если смысла не имеет, значит они считаются недопустимыми.

    Для уточнения вышесказанного: если переменных более одной, тогда может быть и пара подходящих значений.

    Для примера рассмотрим выражение вида 1 x – y + z , где имеются три переменные. Иначе можно записать, как x = 0 , y = 1 , z = 2 , другая же запись имеет вид ( 0 , 1 , 2 ) . Данные значения называют допустимыми, значит, можно найти значение выражения. Получим, что 1 0 – 1 + 2 = 1 1 = 1 . Отсюда видим, что ( 1 , 1 , 2 ) недопустимы. Подстановка дает в результате деление на ноль, то есть 1 1 – 2 + 1 = 1 0 .

    Что такое ОДЗ?

    Область допустимых значений – важный элемент при вычислении алгебраических выражений. Поэтому стоит обратить на это внимание при расчетах.

    Область ОДЗ – это множество значений, допустимых для данного выражения.

    Рассмотрим на примере выражения.

    Если имеем выражение вида 5 z – 3 , тогда ОДЗ имеет вид ( − ∞ , 3 ) ∪ ( 3 , + ∞ ) . Эта область допустимых значений, удовлетворяющая переменной z для заданного выражения.

    Если имеется выражения вида z x – y , тогда видно, что x ≠ y , z принимает любое значение. Это и называют ОДЗ выражения. Его необходимо учитывать, чтобы не получить при подстановке деление на ноль.

    Область допустимых значений и область определения имеет один и тот же смысл. Только второй из них используется для выражений, а первый – для уравнений или неравенств. При помощи ОДЗ выражение или неравенство имеет смысл. Область определения функции совпадает с областью допустимых значений переменной х к выражению f ( x ) .

    Как найти ОДЗ? Примеры, решения

    Найти ОДЗ означает найти все допустимые значения, подходящие для заданной функции или неравенства. При невыполнении этих условий можно получить неверный результат. Для нахождения ОДЗ зачастую необходимо пройти через преобразования в заданном выражении.

    Существуют выражения, где их вычисление невозможно:

    • если имеется деление на ноль;
    • извлечение корня из отрицательного числа;
    • наличие отрицательного целого показателя – только для положительных чисел;
    • вычисление логарифма отрицательного числа;
    • область определения тангенса π 2 + π · k , k ∈ Z и котангенса π · k , k ∈ Z ;
    • нахождение значения арксинуса и арккосинуса числа при значении, не принадлежащем [ – 1 ; 1 ] .

    Все это говорит о том, как важно наличие ОДЗ.

    Найти ОДЗ выражения x 3 + 2 · x · y − 4 .

    Решение

    В куб можно возводить любое число. Данное выражение не имеет дроби, поэтому значения x и у могут быть любыми. То есть ОДЗ – это любое число.

    Ответ: x и y – любые значения.

    Найти ОДЗ выражения 1 3 – x + 1 0 .

    Решение

    Видно, что имеется одна дробь, где в знаменателе ноль. Это говорит о том, что при любом значении х мы получим деление на ноль. Значит, можно сделать вывод о том, что это выражение считается неопределенным, то есть не имеет ОДЗ.

    Ответ: ∅ .

    Найти ОДЗ заданного выражения x + 2 · y + 3 – 5 · x .

    Решение

    Наличие квадратного корня говорит о том, что это выражение обязательно должно быть больше или равно нулю. При отрицательном значении оно не имеет смысла. Значит, необходимо записать неравенство вида x + 2 · y + 3 ≥ 0 . То есть это и есть искомая область допустимых значений.

    Ответ: множество x и y , где x + 2 · y + 3 ≥ 0 .

    Определить ОДЗ выражения вида 1 x + 1 – 1 + log x + 8 ( x 2 + 3 ) .

    Решение

    По условию имеем дробь, поэтому ее знаменатель не должен равняться нулю. Получаем, что x + 1 – 1 ≠ 0 . Подкоренное выражение всегда имеет смысл, когда больше или равно нулю, то есть x + 1 ≥ 0 . Так как имеет логарифм, то его выражение должно быть строго положительным, то есть x 2 + 3 > 0 . Основание логарифма также должно иметь положительное значение и отличное от 1 , тогда добавляем еще условия x + 8 > 0 и x + 8 ≠ 1 . Отсюда следует, что искомое ОДЗ примет вид:

    x + 1 – 1 ≠ 0 , x + 1 ≥ 0 , x 2 + 3 > 0 , x + 8 > 0 , x + 8 ≠ 1

    Иначе говоря, называют системой неравенств с одной переменной. Решение приведет к такой записи ОДЗ [ − 1 , 0 ) ∪ ( 0 , + ∞ ) .

    Ответ: [ − 1 , 0 ) ∪ ( 0 , + ∞ )

    Почему важно учитывать ОДЗ при проведении преобразований?

    При тождественных преобразованиях важно находить ОДЗ. Бывают случаи, когда существование ОДЗ не имеет место. Чтобы понять, имеет ли решение заданное выражение, нужно сравнить ОДЗ переменных исходного выражения и ОДЗ полученного.

    • могут не влиять на ОДЗ;
    • могут привести в расширению или дополнению ОДЗ;
    • могут сузить ОДЗ.

    Рассмотрим на примере.

    Если имеем выражение вида x 2 + x + 3 · x , тогда его ОДЗ определено на всей области определения. Даже при приведении подобных слагаемых и упрощении выражения ОДЗ не меняется.

    Если взять пример выражения x + 3 x − 3 x , то дела обстоят иначе. У нас имеется дробное выражение. А мы знаем, что деление на ноль недопустимо. Тогда ОДЗ имеет вид ( − ∞ , 0 ) ∪ ( 0 , + ∞ ) . Видно, что ноль не является решением, поэтому добавляем его с круглой скобкой.

    Рассмотрим пример с наличием подкоренного выражения.

    Если имеется x – 1 · x – 3 , тогда следует обратить внимание на ОДЗ, так как его необходимо записать в виде неравенства ( x − 1 ) · ( x − 3 ) ≥ 0 . Возможно решение методом интервалов, тогда получаем, что ОДЗ примет вид ( − ∞ , 1 ] ∪ [ 3 , + ∞ ) . После преобразования x – 1 · x – 3 и применения свойства корней имеем, что ОДЗ можно дополнить и записать все в виде системы неравенства вида x – 1 ≥ 0 , x – 3 ≥ 0 . При ее решении получаем, что [ 3 , + ∞ ) . Значит, ОДЗ полностью записывается так: ( − ∞ , 1 ] ∪ [ 3 , + ∞ ) .

    Нужно избегать преобразований, которые сужают ОДЗ.

    Рассмотрим пример выражения x – 1 · x – 3 , когда х = – 1 . При подстановке получим, что – 1 – 1 · – 1 – 3 = 8 = 2 2 . Если это выражение преобразовать и привести к виду x – 1 · x – 3 , тогда при вычислении получим, что 2 – 1 · 2 – 3 выражение смысла не имеет, так как подкоренное выражение не должно быть отрицательным.

    Следует придерживаться тождественных преобразований, которые ОДЗ не изменят.

    Если имеются примеры, которые его расширяют, тогда его нужно добавлять в ОДЗ.

    Рассмотрим на примере дроби вида x x 3 + x . Если сократить на x , тогда получаем, что 1 x 2 + 1 . Тогда ОДЗ расширяется и становится ( − ∞ 0 ) ∪ ( 0 , + ∞ ) . Причем при вычислении уже работаем со второй упрощенной дробью.

    При наличии логарифмов дело обстоит немного иначе.

    Если имеется выражение вида ln x + ln ( x + 3 ) , его заменяют на ln ( x · ( x + 3 ) ) , опираясь на свойство логарифма. Отсюда видно, что ОДЗ с ( 0 , + ∞ ) до ( − ∞ , − 3 ) ∪ ( 0 , + ∞ ) . Поэтому для определения ОДЗ ln ( x · ( x + 3 ) ) необходимо производить вычисления на ОДЗ, то есть ( 0 , + ∞ ) множества.

    При решении всегда необходимо обращать внимание на структуру и вид данного по условию выражения. При правильном нахождении области определения результат будет положительным.

    Как найти область определения функции

    После этого экскурса в важную составную матанализа многие согласятся, что найти область определения функции не очень сложно. Ненамного сложнее, чем Московскую область на карте.

    Во-первых, нужно различать виды функций (корень, дробь, синус и др.). Во-вторых, решать уравнения и неравенства с учетом вида функции (например, на что нельзя делить, какое выражение не может быть под знаком корня и тому подобное). Согласитесь, не так уж много и не так сложно.

    Итак, чтобы находить области определения распространённых функций, порешаем уравнения и неравенства с одной переменной. А в конце урока обобщим понятие на уровне теории. Пока же — краткое определение. Область определения функции y=f(x) — это множество значений X, для которых существуют значения Y.

    Будут и задачи для самостоятельного решения, к которым можно посмотреть ответы.

    Приступаем к практике. На рисунке изображён график функции . Знаменатель дроби не может быть равен нулю, так как на нуль делить нельзя. Поэтому, приравнивая знаменатель нулю, получаем значение, не входящее в область определения функции: 1. То есть, область определения заданной функции — это все значения «икса» от минус бесконечности до единицы и от единицы до плюс бесконечности. Это хорошо видно на графике. Приведённый здесь пример функции относится к виду дробей. На уроке разберём решения всех распространённых видов функций.

    Пример 0. Как найти область определения функции игрек равен квадратному корню из икса минус пять (подкоренное выражение икс минус пять) ()? Нужно всего лишь решить неравенство

    x — 5 ≥ 0,

    так как для того, чтобы мы получили действительное значение игрека, подкоренное выражение должно быть больше или равно нулю. Получаем решение: область определения функции — все значения икса больше или равно пяти (или икс принадлежит промежутку от пяти включительно до плюс бесконечности).

    На чертеже сверху — фрагмент числовой оси. На ней область опредения рассмотренной функции заштрихована, при этом в «плюсовом» направлении штриховка продолжается бесконечно вместе с самой осью.

    Постоянная (константа) определена при любых действительных значениях x, следовательно, данная функция определена на всём множестве R действительных чисел. Это можно записать и так: областью определения данной функции является вся числовая прямая ]- ∞; + ∞[.

    Пример 1. Найти область определения функции y = 2.

    Решение. Область определения функции не указана, значит, в силу выше приведённого определения имеется в виду естественная область определения. Выражение f(x) = 2 определено при любых действительных значениях x, следовательно, данная функция определена на всём множестве R действительных чисел.

    Поэтому на чертеже сверху числовая прямая заштрихована на всём протяжении от минус бесконечности до плюс бесконечности.

    В случае, когда функция задана формулой и n — натуральное число:

    Пример 2. Найти область определения функции .

    Решение. Как следует из определения, корень чётной степени имеет смысл, если подкоренное выражение неотрицательно, то есть, если — 1 ≤ x ≤ 1. Следовательно, область определения данной функции — [- 1; 1].

    Заштрихованная область числовой прямой на чертеже сверху — это область определения данной функции.

    Область определения степенной функции с целым показателем степени

    В случае, когда функция задана формулой :

    если a — положительное, то областью определения функции является множество всех действительных чисел, то есть ]- ∞; + ∞[;

    если a — отрицательное, то областью определения функции является множество ]- ∞; 0[ ∪ ]0 ;+ ∞[, то есть вся числовая прямая за исключением нуля.

    На соответствующем чертеже сверху вся числовая прямая заштрихована, а точка, соответствующая нулю, выколота (она не входит в область определения функции).

    Пример 3. Найти область определения функции .

    Решение. Первое слагаемое целой степенью икса, равной 3, а степень икса во втором слагаемом можно представить в виде единицы — так же целого числа. Следовательно, область определения данной функции — вся числовая прямая, то есть ]- ∞; + ∞[.

    Область определения степенной функции с дробным показателем степени

    В случае, когда функция задана формулой :

    если — положительное, то областью определения функции является множество [0; + ∞[;

    если — отрицательное, то областью определения функции является множество ]0; + ∞[.

    Пример 4. Найти область определения функции .

    Решение. Оба слагаемых в выражении функции — степенные функции с положительными дробными показателями степеней. Следовательно, область определения данной функции — множество [0; + ∞[.

    На чертеже сверху заштрихована часть числовой прямой от нуля (включительно) и больше, причём штриховка продолжается вместе с самой прямой до плюс бесконечности.

    Пример 5. Найти область определения функции .

    Решение. Дробный показатель степени данной степенной функции — отрицательный. Поэтому решим строгое неравенство, когда квадратный трёхчлен в скобках строго больше нуля::

    .

    Дикриминант получился отрицательный. Следовательно сопряжённое неравенству квадратное уравнение не имеет корней. А это значит, что квадратный трёхчлен ни при каких значениях «икса» не равен нулю. Таким образом, область определения данной функции — вся числовая ось, или, что то же самое — множество R действительных чисел, или, что то же самое — ]- ∞; + ∞[.

    Область определения показательной функции

    В случае, когда функция задана формулой , областью определения функции является вся числовая прямая, то есть ]- ∞; + ∞[.

    Область определения логарифмической функции

    Логарифмическая функция определена при условии, если её аргумент положителен, то есть, областью её определения является множество ]0; + ∞[.

    Найти область определения функции самостоятельно, а затем посмотреть решение


    Область определения функции y = cos(x) — так же множество R действительных чисел.

    Область определения функции y = tg(x) — множество R действительных чисел, кроме чисел .

    Область определения функции y = ctg(x) — множество R действительных чисел, кроме чисел .

    Пример 8. Найти область определения функции .

    Решение. Внешняя функция — десятичный логарифм и на область её определения распространяются условия области определения логарифмической функции вообще. То есть, её аргумент должен быть положительным. Аргумент здесь — синус «икса». Поворачивая воображаемый циркуль по окружности, видим, что условие sin x > 0 нарушается при «иксе» равным нулю, «пи», два, умноженном на «пи» и вообще равным произведению числа «пи» и любого чётного или нечётного целого числа.

    Таким образом, область определения данной функции задаётся выражением

    ,

    где k — целое число.

    Область определения обратных тригонометрических функций

    Область определения функции y = arcsin(x) — множество [-1; 1].

    Область определения функции y = arccos(x) — так же множество [-1; 1].

    Область определения функции y = arctg(x) — множество R действительных чисел.

    Область определения функции y = arcctg(x) — так же множество R действительных чисел.

    Пример 9. Найти область определения функции .

    Решение. Решим неравенство:

    Таким образом, получаем область определения данной функции — отрезок [- 4; 4].

    Пример 10. Найти область определения функции .

    Решение. Решим два неравенства:

    Решение первого неравенства:

    Решение второго неравенства:

    Таким образом, получаем область определения данной функции — отрезок [0; 1].

    Если функция задана дробным выражением, в котором переменная находится в знаменателе дроби, то областью определения функции является множество R действительных чисел, кроме таких x, при которых знаменатель дроби обращается в нуль.

    Пример 11. Найти область определения функции .

    Решение. Решая равенство нулю знаменателя дроби, находим область определения данной функции — множество ]- ∞; — 2[ ∪ ]- 2 ;+ ∞[.

    Пример 12. Найти область определения функции .

    Решение. Решим уравнение:

    Таким образом, получаем область определения данной функции — ]- ∞; — 1[ ∪ ]- 1 ; 1[ ∪ ]1 ;+ ∞[.

    Пример 13. Найти область определения функции .

    Решение. Область определения первого слагаемого — данной функции — множество R действительных чисел, второго слагаемого — все действительные числа, кроме -2 и 2 (получили, решая равенство нулю знаменателя, как в предыдущем примере). В этом случае область определения функции должна удовлетворять условиями определения обоих слагаемых. Следовательно, область определения данной функции — все x, кроме -2 и 2.

    Пример 14. Найти область определения функции .

    Решение. Решим уравнение:

    Уравнение не имеет действительных корней. Но функция определена только на действительных числах. Таким образом, получаем область определения данной функции — вся числовая прямая или, что то же самое — множество R действительных чисел или, что то же самое — ]- ∞; + ∞[.

    То есть, какое бы число мы не подставляли вместо «икса», знаменатель никогда не будет равен нулю.

    Пример 15. Найти область определения функции .

    Решение. Решим уравнение:

    Таким образом, получаем область определения данной функции — ]- ∞; — 1[ ∪ ]- 1 ; 0[ ∪ ]0 ; 1[ ∪ ]1 ;+ ∞[.

    Пример 16. Найти область определения функции .

    Решение. Кроме того, что знаменатель не может быть равным нулю, ещё и выражение под корнем не может быть отрицательным. Сначала решим уравнение:

    График квадратичной функции под корнем представляет собой параболу, ветви которой направлены вверх. Как следует из решения квадратного уравнения, парабола пересекает ось Ox в точках 1 и 2. Между этими точками линия параболы находится ниже оси Ox, следовательно значения квадратичной функции между этими точками отрицательное. Таким образом, исходная функция не определена на отрезке [1; 2].

    Найти область определения функции самостоятельно, а затем посмотреть решение


    Если функция задана формулой вида y = kx + b, то область определения функции — множество R действительных чисел.

    А теперь обобщим решения рассмотренных примеров. Каждой точке графика функции соответствуют:

    • определённое значение «икса» — аргумента функции;
    • определённое значение «игрека» — самой функции.
    Верны следующие факты.
    • От аргумента — «икса» — вычисляется «игрек» — значения функции.
    • Область определения функции — это множества всех значений «икса», для которых существует, то есть может быть вычислен «игрек» — значение функции. Иначе говоря, множество значений аргумента, на котором «функция работает».

    Весь раздел «Исследование функций»

    Реферат по математике «Область определения и область значений функции»

    Область определения и область значений функции. 

    Определение:
    Множество первых элементов пары  f f   называется   областью определения , а множество вторых элементов называется   множеством значений . Область определения и множество значений функции обозначаются как   D f ​   и   R f ​   соответственно.

    Функция  y= f(x) y = f ( x )   — это такая зависимость переменной   y  от переменной  x , когда каждому допустимому значению переменной   x  соответствует единственное значение переменной  y .

    Областью определения функции  D(f) D ( f )   называют множество всех допустимых значений переменной  x .

    Область значений функции  E(f) E ( f )   — множество всех допустимых значений переменной  y .

    Область определения функции

    Начнем с исследования области определения и области значений функций. Вспомним, что  областью определения функции  называют все возможные значения аргумента    (мы говорим о естественной области определения). Обычно область определения обозначают как   . Пока что мы знаем только две недопустимые операции – это деление на ноль и извлечение квадратного корня из отрицательного числа.

    Поэтому при нахождении области определения функции ограничения появляются в двух случаях.

    1. В функции есть деление на переменные. В этом случае приравниваем знаменатель дроби к нулю. Решая полученное уравнение, получаем  недопустимые   значения аргумента . Тогда областью определения будут все действительные числа, кроме недопустимых значений.

    2. В функции есть операция извлечения корня. Тогда подкоренное выражение должно быть неотрицательным. Записываем соответствующее неравенство. Множество решений этого неравенства и будет областью определения функции.

    Краткая запись промежутков

    Область определения мы смогли описать словами: все действительные числа, кроме  . Но словесное описание в математике редко встречается, ведь обычно оно получается громоздким. Поэтому вводят специальные обозначения.

    Так, если мы хотим указать на множество чисел, лежащих в некотором промежутке, то выполняем следующие действия.

    1. Через точку с запятой указываем два числа: левую и правую границы промежутка.

    2. Если граница входит в промежуток, ставим возле нее квадратную скобку, если не входит – круглую.

    3. Если у промежутка нет правой границы, записываем ее как   (или   ). Если нет левой границы, пишем   .

    4. Если нужно описать множество, состоящее из нескольких промежутков, ставим между ними знак объединения:   .

    Например, все действительные числа от    до    включительно можно записать так:   . Все положительные числа можно описать как  . Ноль не является положительным числом, поэтому скобка возле него круглая, возле бесконечности скобка всегда круглая.

    В примере с областью определения мы получили два промежутка: все числа, большие   и все числа, меньшие   . Поэтому и записали два соответствующих промежутка   , поставив между ними знак объединения.

    Область значений функции

    Область (или множество) значений функции  – это все возможные значения   . Область значений принято обозначать   .

    Вспомним графики базовых функций и их области значений:

    1. Линейная функция — это функция вида  y=kx+b y = k x + b , где  k  и  b  некоторые действительные числа. Если  b=0 b = 0 , то функция примет вид   y=kx y = k x   и будет называться   прямой пропорциональностью .x, где   a=const, a > 0, , a ≠ 1 D(f) : x ∈ R ; E ( f ) : y ∈ ( 0 ; + ∞ ) . Графиком показательной функции является экспонента.

    2. Логарифмическая функция;

    3. Тригонометрическая функция;

    4. Обратные тригонометрические функции;

    Область и диапазон функции

    Определения домена и диапазона

    Домен

    Домен а функция — это полный набор возможных значений независимой переменной.

    На простом английском языке это определение означает:

    Домен — это совокупность всех возможных x — значения, которые сделают функцию «работа» и выдаст реальные значения и .

    При нахождении домена запомните:

    • Знаменатель (внизу) дроби не может быть ноль
    • Число под знаком квадратного корня должно быть положительный в этом разделе

    Пример 1а

    Вот график y = sqrt (x + 4):

    12345-1-2-3-4123xy

    Домен: `x> = — 4`

    Область определения этой функции — `x ≥ −4`, так как x не может быть меньше, чем` −4`.Чтобы понять, почему, попробуйте в вашем калькуляторе некоторые числа меньше, чем «−4» (например, «−5» или «−10»), и некоторые числа больше, чем «−4» (например, «−2» или «8»). Единственные, которые «работают» и дают нам ответ, — это те, которые больше или равны «−4». Это сделает число под квадратным корнем положительным.

    Примечания:

    1. Закрашенный кружок в точке `(-4, 0)`. Это указывает на то, что домен «запускается» в этот момент.
    2. Мы видели, как рисовать подобные графики в разделе 4, График функции.2 = х — 2.

    Как найти домен

    В общем, мы определяем область каждой функции, ища те значения независимой переменной (обычно x ), которые разрешено использовать . (Обычно нам нужно избегать 0 в нижней части дроби или отрицательных значений под знаком квадратного корня).

    Диапазон

    Диапазон из функция — это полный набор всех возможных результирующих значений зависимой переменной ( y, обычно ) после того, как мы подставили домен.

    На простом английском языке это определение означает:

    Диапазон — это результат y — значения , которые мы получаем после подстановки всех возможных значений x .

    Как найти диапазон

    • Диапазон функции — это разброс возможных значений y (от минимального y -значения до максимального y -значения)
    • Подставьте различные значения x в выражение для y на посмотреть, что происходит.(Спросите себя: всегда ли и положительны? Всегда отрицательны? Или, может быть, не равны определенным значениям?)
    • Убедитесь, что вы ищете минимум и максимум значений y .
    • Нарисуйте эскиз ! В математике картина стоит тысячи слов.

    Пример 1б

    Вернемся к примеру выше, `y = sqrt (x + 4)`.

    Мы замечаем, что кривая находится либо на горизонтальной оси, либо над ней.Независимо от того, какое значение x мы попробуем, мы всегда получим нулевое или положительное значение y . Мы говорим, что диапазон в этом случае равен y ≥ 0.

    12345-1-2-3-4123xy

    Диапазон: `y> = 0`

    Кривая всегда продолжается вертикально, за пределы того, что показано на графике, поэтому диапазон — это все неотрицательные значения `y`.

    Пример 2

    График кривой y = sin x показывает диапазон между -1 и 1.

    12345-1-2-3-4-5-6-71-1xy

    Диапазон: `-1

    Область y = sin x — это «все значения x », поскольку нет никаких ограничений на значения для x . (Введите любое число в функцию «sin» в вашем калькуляторе. Любое число должно работать и даст вам окончательный ответ от -1 до 1.)

    Эксперимент с калькулятором и наблюдение кривой показывают, что диапазон составляет y между -1 и 1.Мы могли бы записать это как −1 ≤ y ≤ 1.

    Откуда взялся этот график? Мы узнаем о графиках sin и cos позже в Графах греха x и cos x

    Примечание 1: Поскольку мы предполагаем, что для значений x должны использоваться только действительные числа, числа, которые приводят к делению на ноль или к мнимым числам (которые возникают при нахождении квадратного корня из отрицательное число) не включаются.В главе «Комплексные числа» подробно рассказывается о мнимых числах, но мы не включаем такие числа в эту главу.

    Примечание 2: При выполнении примеров квадратного корня многие люди спрашивают: «Разве мы не получаем 2 ответа, один положительный и один отрицательный, когда мы находим квадратный корень?» Квадратный корень имеет не более одного значения, а не два. См. Это обсуждение: Квадратный корень 16 — сколько ответов?

    Примечание 3: Мы говорим о домене и диапазоне функций , которые имеют не более , одно значение y для каждого значения x , а не отношений (которые могут иметь более одного . 2-9),` без использования графика.2-9`, которое, как мы понимаем, можно записать как `(x + 3) (x-3)`. Таким образом, наши значения для `x` не могут включать` -3` (из первой скобки) или `3` (из второй).

    В любом случае нам не нужно беспокоиться о «-3», потому что на первом этапе мы решили, что «x> = -2».

    Таким образом, домен для этого случая — `x> = -2, x! = 3`, который мы можем записать как` [-2,3) uu (3, oo) `.

    Для определения диапазона мы рассматриваем верхнюю и нижнюю части дроби отдельно.

    Числитель: Если `x = -2`, верхняя часть имеет значение` sqrt (2 + 2) = sqrt (0) = 0`.2-9) `приближается к` 0`, поэтому `f (x)` переходит в `-oo`, когда приближается к` x = 3`.

    Для `x> 3`, когда` x` просто больше, чем `3`, значение дна чуть больше` 0`, поэтому `f (x)` будет очень большим положительным числом.

    Для очень большого `x` верхняя часть будет большой, но нижняя будет намного больше, поэтому в целом значение функции будет очень маленьким.

    Таким образом, мы можем заключить, что диапазон равен `(-oo, 0] uu (oo, 0)`.

    Посмотрите на график (который мы все равно рисуем, чтобы убедиться, что мы на правильном пути):

    Показать график

    Мы можем видеть на следующем графике, что действительно домен равен «[-2,3) uu (3, oo)» (который включает «-2», но не «3»), а диапазон — «все значения из `f (x)`, кроме `F (x) = 0`.2-9) `.

    Сводка

    В общем, мы определяем домен по ищем те значения независимой переменной (обычно x ), которые у нас разрешено использовать . (Мы должны избегать 0 в нижней части дроби или отрицательных значений под знаком квадратного корня).

    Диапазон находится путем нахождения результирующих значений y после замены возможных значений x .

    Упражнение 1

    Найдите домен и диапазон для каждого из следующих.2+ 2`.

    Ответ

    Домен: Функция

    f ( x ) = x 2 + 2

    определен для всех реальных значений x (поскольку нет ограничений на значение x ).

    Следовательно, область `f (x)` равна

    «все реальные значения x «.

    Диапазон: Поскольку x 2 никогда не бывает отрицательным, x 2 + 2 никогда не меньше `2`

    Следовательно, диапазон `f (x)` равен

    «все действительные числа` f (x) ≥ 2` «.

    Мы видим, что x может принимать любое значение на графике, но результирующие значения y = f ( x ) больше или равны 2.

    123-1-2-312345678910-1xf (x)

    Диапазон: `y> = 2`

    Домен: Все `x`

    Примечание

    1. При построении графиков важно обозначить оси как . Это помогает понять, что представляет собой график.
    2. Мы видели, как рисовать такие графики в Графике функции.

    (б) `f (t) = 1 / (t + 2)`

    Ответ

    Домен: Функция

    `f (t) = 1 / (t + 2)`

    не определено для т = -2, так как это значение приведет к делению на ноль. (Внизу дроби будет 0.)

    Следовательно, домен из f ( t ) равен

    «все вещественные числа кроме -2 «

    Диапазон: Независимо от того, насколько большим или малым становится т , f ( t ) никогда не будет равно нулю.

    [ Почему? Если мы попытаемся решить уравнение относительно 0, произойдет следующее:

    `0 = 1 / (t + 2)`

    Умножаем обе стороны на ( t + 2) и получаем

    `0 = 1`

    Это невозможно.]

    Таким образом, диапазон для f ( t ) равен

    «все вещественные числа кроме нуля ».

    На графике видно, что функция не определена для t = -2 и что функция (значения y ) принимает все значения, кроме 0.

    1234-1-2-3-4-5-6-712345-1-2-3-4-5tf (t)

    Домен: Все `t ≠ -2`

    Диапазон: Все `f (t) ≠ 0`

    (c) `g (s) = sqrt (3-s)`

    Ответ

    Функция

    `г (т) = sqrt (3-с)`

    не определен для реального числа больше 3, что приведет к мнимым значениям для г ( с ). 2 + 4` для `x> 2`

    Ответ

    Функция `f (x)` имеет область из «все действительные числа,` x> 2` «, как определено в вопросе.(Здесь не используются квадратные корни из отрицательных чисел или деления на ноль.)

    Чтобы найти диапазон :

    • Когда `x = 2`,` f (2) = 8`
    • Когда x увеличивается с `2`,` f (x) `становится больше, чем `8` (попробуйте подставить некоторые числа, чтобы понять, почему)

    Следовательно, диапазон — это «все действительные числа,` f (x)> 8` »

    Вот график функции с белым кружком в «(2, 8)», что указывает на то, что домен не включает «x = 2», а диапазон не включает «f (2) = 8».

    123456510152025xf (x) (2, 8)

    Домен: Все `x> 2`

    Диапазон:
    Все `f (x)> 8`

    Функция является частью параболы. [Подробнее о параболе.]

    Упражнение 2

    Мы запускаем шар в воздух и находим высота h , в метрах, как функция времени т , в секундах, равно

    ч = 20 т — 4,9 т 2

    Найдите домен и диапазон для функции ч ( т ).

    Ответ

    Как правило, отрицательные значения времени не имеют имея в виду. Кроме того, нам нужно предположить, что снаряд попадает в землю, а затем останавливается — он не уходит под землю.

    Итак, нам нужно рассчитать, когда он упадет на землю. Это будет, когда h = 0. Итак, решаем:

    20 т — 4,9 т 2 = 0

    Факторинг дает:

    (20 — 4.9 т ) т = 0

    Это верно, когда

    `t = 0 \» s «`,

    или

    `t = 20/4.9 = 4.082 текст (ы) `

    Следовательно, домен функции h равен

    «все реально значения t такие, что `0 ≤ t ≤ 4.082`»

    Из выражения функции видно, что это парабола с вершиной вверх. (Это имеет смысл, если вы думаете о подбрасывании мяча вверх. Он поднимается на определенную высоту, а затем падает обратно.)

    Какое максимальное значение ч ? Воспользуемся формулой максимума (или минимума) квадратичной функции.

    Значение т. дает максимальное значение

    .

    `t = -b / (2a) = -20 / (2 xx (-4.9)) = 2.041 с`

    Таким образом, максимальное значение —

    .

    20 (2,041) — 4,9 (2,041) 2 = 20,408 м

    Наблюдая за функцией h , мы видим, что по мере увеличения t , h сначала увеличивается до максимума. 20,408 м, затем ч снова уменьшается до нуля, как и ожидалось.

    Следовательно, диапазон h равен

    «все реально числа, `0 ≤ h ≤ 20,408`»

    Вот график функции h :

    1234565101520-5-й (t)

    Домен: `0

    Диапазон:
    `0

    Функции, определяемые координатами

    Иногда у нас нет непрерывных функций. Что нам делать в этом случае? Давайте посмотрим на пример.

    Упражнение 3

    Найдите область и диапазон функции, заданной координатами:

    `{(−4, 1), (−2, 2.5), (2, −1), (3, 2)} `

    Ответ

    Область — это просто следующие значения x : `x = {−4, −2, 2, 3}`

    Диапазон состоит из следующих значений `f (x)`: `f (x) = {−1, 1, 2, 2.5}`

    Вот график нашей разрывной функции.

    1234-1-2-3-41234-1-2-3-е (т) (3, 2) (2, -1) (- 4, 1)

    (-2, 2,5)

    Область и диапазон функции — объяснение и примеры

    В этой статье объясняется область и диапазон среднего значения функции, а также способы вычисления двух величин. Прежде чем перейти к теме домена и диапазона, давайте кратко опишем, что такое функция.

    В математике мы можем сравнить функцию с машиной, которая генерирует некоторый результат в корреляции с заданным входом . На примере машины для чеканки монет мы можем проиллюстрировать значение функции следующим образом.

    Когда вы вставляете монету в монетоприемник, в результате получается штампованный и сплющенный кусок металла. Рассматривая функцию, мы можем связать монету и сплющенный кусок металла с доменом и диапазоном.В этом случае функцией считается машина для чеканки монет.

    Так же, как станок для штамповки монет, который может производить только один сплющенный кусок металла за один раз, функция работает таким же образом, выдавая один результат за один раз.

    История функции

    Идея функции появилась в начале семнадцатого века, когда Рене Декарт (1596-1650) использовал эту концепцию в своей книге «Геометрия» (1637) для моделирования математических задач.

    Пятьдесят лет спустя, после публикации «Геометрии», Готфрид Вильгельм Лейбниц (1646-1716) ввел термин «функция». Позже Леонард Эйлер (1707-1783) сыграл большую роль, введя технику понятия функции y = f (x).

    Реальное применение функции

    Функции очень полезны в математике, потому что они позволяют нам моделировать реальные проблемы в математическом формате.

    Вот несколько примеров применения функции.
    • Окружность окружности

    Окружность окружности зависит от ее диаметра или радиуса. Мы можем математически представить это утверждение как:

    C (d) = dπ или C (r) = 2π⋅r

    Длина тени объекта является функцией его высоты.

    • Положение движущегося объекта

    Местоположение движущегося объекта, например автомобиля, зависит от времени.

    Температура тела зависит от нескольких факторов и входных данных.

    Сложный или простой процент зависит от времени, основной суммы и процентной ставки.

    Высота объекта зависит от его возраста и массы тела.

    Теперь, узнав о функции, можно перейти к вычислению области и диапазона функции.

    Какова область и диапазон функции?

    Область функции — это входные числа, которые при подключении к функции определяют результат. Проще говоря, мы можем определить область определения функции как возможные значения x, которые сделают уравнение истинным.

    Некоторые из случаев, когда функция не может быть действительной, — это когда уравнение делится на ноль или отрицательный квадратный корень.

    Например, f ( x ) = x 2 является допустимой функцией, потому что независимо от того, какое значение x можно подставить в уравнение, всегда есть правильный ответ. По этой причине мы можем заключить, что область определения любой функции — это действительные числа.

    Диапазон функции определяется как набор решений уравнения для заданного входа.Другими словами, диапазон — это результат или значение y функции. Для данной функции существует только один диапазон.

    Как использовать обозначения интервалов для указания домена и диапазона?

    Поскольку диапазон и область определения функции обычно выражаются в интервальной записи, важно обсудить концепцию интервальной записи.

    Процедура записи интервалов включает:

    • Запишите числа, разделенные запятой, в порядке возрастания.
    • Заключите числа в круглые скобки (), чтобы показать, что значение конечной точки не включено.
    • Используйте квадратные скобки [], чтобы заключить числа, когда включено значение конечной точки.

    Как найти домен и диапазон функции?

    Мы можем определить область определения функции либо алгебраически, либо графическим методом. Чтобы вычислить область определения функции алгебраически, вы решаете уравнение, чтобы определить значения x.

    У разных типов функций есть свои методы определения своей области.

    Давайте рассмотрим эти типы функций и способы вычисления их области.

    Как найти область определения функции без знаменателя и радикалов?

    Давайте рассмотрим несколько примеров ниже, чтобы понять этот сценарий.

    Пример 1

    Найдите область определения f (x) = 5x — 3

    Решение

    Областью определения линейной функции являются все действительные числа, поэтому

    Область: (−∞, ∞)

    Диапазон: (−∞, ∞)

    Функция с радикалом

    Пример 2

    Найти область определения функции f (x) = — 2x 2 + 12x + 5

    Решение

    Функция f (x) = −2x 2 + 12x + 5 является квадратичным многочленом, поэтому область определения (−∞, ∞)

    Как найти область для рационального функция с переменной в знаменателе?

    Чтобы найти область определения функции этого типа, установите знаменатель на ноль и вычислите значение переменной.

    Давайте рассмотрим несколько примеров ниже, чтобы понять этот сценарий.

    Пример 3

    Определите область значений x − 4 / (x 2 −2x − 15)

    Решение

    Установите знаменатель равным нулю и решите относительно x

    ⟹ x 2 — 2x — 15 = (x — 5) (x + 3) = 0

    Следовательно, x = −3, x = 5

    Чтобы знаменатель не был равен нулю, нам нужно избегать чисел −3 и 5 Таким образом, домен состоит из действительных чисел, кроме −3 и 5.

    Пример 4

    Вычислить область и диапазон функции f (x) = -2 / x.

    Решение

    Установите знаменатель на ноль.

    ⟹ x = 0

    Следовательно, домен: все действительные числа, кроме 0.

    Диапазон — это все действительные значения x, кроме 0.

    Пример 5

    Найдите домен и диапазон следующей функции .

    f (x) = 2 / (x + 1)

    Решение

    Установите знаменатель равным нулю и решите относительно x.

    x + 1 = 0

    = -1

    Поскольку функция не определена, когда x = -1, доменом являются все действительные числа, кроме -1. Точно так же диапазон — это все действительные числа, кроме 0

    Как определить домен для функции с переменной внутри знака корня?

    Для нахождения области определения функции членам внутри радикала задается неравенство> 0 или ≥ 0. Затем определяется значение переменной.

    Давайте рассмотрим несколько примеров ниже, чтобы понять этот сценарий.

    Пример 6

    Найдите область определения f (x) = √ (6 + x — x 2 )

    Решение

    Чтобы избежать квадратных корней из отрицательных чисел, мы задаем выражение внутри знака корня до ≥ 0.

    6 + x — x 2 ≥ 0 ⟹ x 2 — x — 6≤ 0

    ⟹ x 2 — x — 6 = (x — 3) (x +2) = 0

    Следовательно, функция равна нулю, если x = 3 или x = -2

    Следовательно, домен: [−2, 3]

    Пример 7

    Найдите область определения f ( x) = x / √ (x 2 — 9)

    Решение

    Установите выражение внутри знака радикала равным x 2 — 9> 0
    Найдите переменную, чтобы получить;

    x = 3 или — 3

    Следовательно, домен: (−∞, −3) & (3, ∞)

    Пример 8

    Найдите область определения f (x) = 1 / √ ( x 2 -4)

    Решение

    Разложив знаменатель на множители, мы получим x ≠ (2, — 2).

    Проверьте свой ответ, подставив -3 в выражение внутри знака корня.

    ⟹ (-3) 2 — 4 = 5

    также попробуйте с нулем

    ⟹ 0 2 — 4 = -4, поэтому числа от 2 до -2 недействительны

    Попробуйте число больше 2

    ⟹ 3 2 — 4 = 5. Это верно.

    Следовательно, домен = (-∞, -2) U (2, ∞)

    Как найти область определения функции с помощью натурального логарифма (ln)?

    Чтобы найти домен функции с использованием натурального логарифма, установите для членов в круглых скобках значение> 0 и затем решите.

    Давайте посмотрим на пример ниже, чтобы понять этот сценарий.

    Пример 9

    Найти область определения функции f (x) = ln (x — 8)

    Решение

    ⟹ x — 8> 0

    ⟹ x — 8 + 8> 0 + 8

    ⟹ x> 8

    Домен: (8, ∞)

    Как найти домен и диапазон отношения?

    Отношение — это актив координат x и y. Чтобы найти домен и диапазон в отношении, просто укажите значения x и y соответственно.

    Давайте рассмотрим несколько примеров ниже, чтобы понять этот сценарий.

    Пример 10

    Укажите область и диапазон отношения {(2, –3), (4, 6), (3, –1), (6, 6), (2, 3) }

    Решение

    Перечислите значения x. Домен: {2, 3, 4, 6}

    Список значений y. диапазон: {–3, –1, 3, 6}

    Пример 11

    Найдите домен и диапазон отношения {(–3, 5), (–2, 5), (–1, 5), (0, 5), (1, 5), (2, 5)}

    Решение

    Домен {–3, –2, –1, 0, 1, 2} и диапазон равно {5}

    Пример 12

    Учитывая, что R = {(4, 2) (4, -2), (9, 3) (9, -3)}, найдите домен и диапазон Р.

    Решение

    Домен представляет собой список первых значений, поэтому D = {4, 9} и диапазон = {2, -2, 3, -3}

    Предыдущий урок | Главная страница | Следующий урок

    Домен и диапазон

    В домен из функция ж ( Икс ) — это набор всех значений, для которых определена функция, а диапазон функции — это набор всех значений, которые ж берет.

    (В гимназии вы, вероятно, называли домен набором замены, а диапазон — набором решений. Их также можно было назвать входом и выходом функции.)

    Пример 1:

    Рассмотрим функцию, показанную на диаграмме.

    Здесь домен — это множество { А , B , C , E } .D не входит в домен, так как функция не определена для D .

    Диапазон — это набор { 1 , 3 , 4 } . 2 не входит в диапазон, так как в домене нет буквы, которая сопоставляется с 2 .

    Вы также можете поговорить о домене связь , где один элемент в домене может быть сопоставлен более чем с одним элементом в диапазоне.

    Пример 2:

    Рассмотрим соотношение { ( 0 , 7 ) , ( 0 , 8 ) , ( 1 , 7 ) , ( 1 , 8 ) , ( 1 , 9 ) , ( 2 , 10 ) } .

    Здесь отношение задано как набор упорядоченных пар. Домен — это набор Икс -координаты, { 0 , 1 , 2 } , а диапазон — это набор у -координаты, { 7 , 8 , 9 , 10 } . Обратите внимание, что элементы домена 1 а также 2 связаны с более чем одним элементом диапазона, поэтому это нет функция.

    Но чаще, особенно при работе с графиками на координатной плоскости, мы имеем дело с функциями, в которых каждый элемент области связан с одним элементом диапазона. (См. Тест вертикальной линии .)

    Пример 3:

    Область определения функции

    ж ( Икс ) знак равно 1 Икс

    все действительные числа, кроме нуля (так как at Икс знак равно 0 , функция не определена: деление на ноль недопустимо!).

    Диапазон также состоит из действительных чисел, кроме нуля. Вы можете видеть, что на кривой есть точка для каждого у -значение кроме у знак равно 0 .

    Домены также могут быть указаны явно, если есть значения, для которых функция может быть определена, но которые мы не хотим рассматривать по какой-то причине.

    Пример 4:

    Следующие обозначения показывают, что область определения функции ограничена интервалом ( — 1 , 1 ) .

    ж ( Икс ) знак равно Икс 2 , — 1 < Икс < 1

    График этой функции показан на рисунке. Обратите внимание на белые кружки, которые показывают, что функция не определена в Икс знак равно — 1 а также Икс знак равно 1 . В у -значения варьируются от 0 вплоть до 1 (в том числе 0 , но не включая 1 ).Таким образом, диапазон функции

    0 ≤ у < 1 .

    Домен и Диапазон | Безграничная алгебра

    Введение в домен и диапазон

    Область функции — это набор всех возможных входных значений, которые производят некоторый диапазон выходных значений

    Цели обучения

    Определите область и диапазон функции

    Ключевые выводы

    Ключевые моменты
    • Для данной функции [latex] f [/ latex] набор значений [latex] x [/ latex] (входов) является доменом [latex] f [/ latex], а набор [latex] y [/ latex] значения (выходы) — это диапазон [latex] f [/ latex].
    • Область определения функции [latex] f [/ latex] — это все значения, для которых функция определена. Например, [latex] \ frac {1} {x} [/ latex] не определяется, если [latex] x = 0 [/ latex]. Кроме того, [latex] \ sqrt {x} [/ latex] не определяется, если [latex] x [/ latex] отрицательно.
    • Чтобы найти домен функции [latex] f [/ latex], вы должны найти значения, для которых [latex] f [/ latex] не определено. Итак, домен для [latex] \ sqrt {x} [/ latex] — [latex] x \ geq 0 [/ latex].
    Ключевые термины
    • домен : набор всех точек, по которым определяется функция.
    • диапазон : набор значений, которые функция принимает в качестве выходных данных.
    • функция : отношение между двумя величинами, называемыми входом и выходом; для каждого входа есть ровно один выход.

    Каковы область и диапазон функции?

    Область функции — это набор входных значений [latex] x [/ latex], для которых определена функция. Домен показан в левом овале на картинке ниже. Функция предоставляет выходное значение [latex] f (x) [/ latex] для каждого члена домена.Набор значений, которые выводит функция, называется диапазоном функции, и эти значения показаны в правом овале на рисунке ниже. Функция — это отношение, которое принимает входные данные домена и выводит значения в диапазоне. Правило для функции состоит в том, что для каждого входа есть ровно один выход.

    Отображение функции: Овал слева — это домен функции [latex] f [/ latex], а овал справа — это диапазон.Зеленые стрелки показывают, как каждый член домена сопоставляется с определенным значением диапазона.

    Как вы можете видеть на иллюстрации, у каждого значения домена есть зеленая стрелка, указывающая ровно одно значение диапазона. Следовательно, это отображение является функцией.

    По набору упорядоченных пар, заданных в этом отображении, мы также можем сказать, что это функция, потому что ни одно из значений [latex] x [/ latex] не повторяется: [latex] (- 1,1), (1,1 ), (7,49), (0,5,0,25) [/ латекс]; поскольку каждый вход соответствует ровно одному выходу.(Обратите внимание, что хотя выходное значение [latex] 1 [/ latex] повторяется, только входные значения не могут повторяться)

    Мы также можем сказать это отображение, и набор упорядоченных пар является функцией, основанной на графике упорядоченных пар, потому что точки не образуют вертикальную линию. Если бы значение [latex] x [/ latex] повторялось, были бы две точки, составляющие график вертикальной линии, что не было бы функцией. Давайте посмотрим на это отображение и список упорядоченных пар, построенных на декартовой плоскости.

    Упорядоченные пары: Это отображение или набор упорядоченных пар является функцией, потому что точки не образуют вертикальную линию.2 [/ латекс].

    Важно отметить, что не все функции имеют набор действительных чисел в качестве своей области. Например, функция [latex] f (x) = \ frac {1} {x} [/ latex] не определена для [latex] x = 0 [/ latex], потому что вы не можете разделить число на [latex] 0 [/ латекс]. В этом случае домен [latex] f [/ latex] представляет собой набор всех действительных чисел , кроме [latex] 0 [/ latex]. То есть [латекс] х \ neq0 [/ латекс]. Таким образом, область определения этой функции — [latex] \ mathbb {R} — \ {0 \} [/ latex].

    А как насчет функции [латекс] f (x) = \ sqrt {x} [/ latex]? В этом случае квадратный корень из отрицательного числа не определен, поэтому домен представляет собой набор всех действительных чисел, где [latex] x \ geq0 [/ latex].

    Поиск домена и диапазона: задана функция

    Чтобы найти домен функции, если он не указан с самого начала, нам нужно посмотреть на определение функции, чтобы определить, какие значения не разрешены. Например, мы знаем, что нельзя извлечь квадратный корень из отрицательного числа и нельзя разделить на [латекс] 0 [/ латекс]. Обладая этими знаниями, давайте найдем область определения функции.

    Пример 1: Найдите домен:

    [латекс] \ displaystyle f (x) = \ frac {1} {\ sqrt {x-1} -2} + x [/ latex]

    Во-первых, мы знаем, что не можем разделить на [latex] 0 [/ latex], поэтому любое значение [latex] x [/ latex], которое вызывает деление на [latex] 0 [/ latex], не допускается в домене.В этом примере это происходит, когда:

    [латекс] \ displaystyle \ sqrt {x-1} -2 = 0 [/ латекс]

    Решая для [latex] x [/ latex], это происходит, когда [latex] x = 5 [/ latex], поэтому мы знаем, что [latex] x \ neq5 [/ latex].

    Мы также знаем, что нельзя извлекать квадратный корень из отрицательного числа. Это означает, что:

    [латекс] \ displaystyle x-1> 0 [/ латекс]

    После решения для [latex] x [/ latex] мы видим, что [latex] x> 1 [/ latex]. Таким образом, домен этой функции — это набор всех действительных чисел, таких что [latex] x> 1 [/ latex] и [latex] x \ neq5 [/ latex].

    Следовательно, чтобы найти, какие значения отсутствуют в домене, вы должны найти значения, в которых функция не определена.

    Визуализация области и диапазона

    Все значения в домене отображаются на значения в диапазоне, которые отображаются в виде графиков функций

    Цели обучения

    Используйте график функции, чтобы определить ее домен и диапазон

    Ключевые выводы

    Ключевые моменты
    • Значения в домене отображаются на значения в диапазоне.
    • Тест горизонтальной и вертикальной линии может помочь определить тип связи между доменом и диапазоном.
    Ключевые термины
    • диапазон : Набор значений (точек), которые может получить функция.
    • домен : набор всех точек, по которым определяется функция.
    • функция : любая математическая формула, которая дает один и только один результат для каждого ввода.

    Обзор домена, диапазона и функций

    Как указано в предыдущем разделе, область функции — это набор «входных» значений [latex] (x) [/ latex], для которых функция определена.{2} [/ latex] имеет диапазон [latex] f (x) \ geq0 [/ latex], потому что квадрат числа всегда дает положительный результат.

    С учетом как области, так и диапазона, функция — это любая математическая формула, которая дает один и только один результат для каждого ввода. Следовательно, каждое заданное значение домена в результате имеет одно и только одно значение диапазона, но не обязательно наоборот. Другими словами, два разных значения [latex] x [/ latex] могут иметь одинаковое значение [latex] y [/ latex], но каждое значение [latex] y [/ latex] должно быть объединено с отдельным [ латекс] х [/ латекс] -значение.3 [/ латекс].

    Пример 1: Определите область и диапазон каждого графика, изображенного ниже:

    Оба графика включают все действительные числа [latex] x [/ latex] в качестве входных значений, поскольку оба графика продолжают влево (отрицательные значения) и вправо (положительные значения) для [latex] x [/ latex] (входные данные) . Кривые уходят в бесконечность в обоих направлениях; поэтому мы говорим, что домен для обоих графов — это набор всех действительных чисел, обозначенных как: [latex] \ mathbb {R} [/ latex].

    Если теперь мы посмотрим на возможные выходы или значения [latex] y [/ latex], [latex] f (x) [/ latex], (глядя вверх и вниз по оси [latex] y [/ latex], обратите внимание, что красный график НЕ включает отрицательные значения [latex] y [/ latex], тогда как синий график включает как положительные, так и отрицательные значения.3 [/ latex] (синий), поскольку все действительные числа могут быть входными значениями. Однако диапазон красного графика ограничен только значениями [latex] f (x) \ geq0 [/ latex] или [latex] y [/ latex] выше или равными [latex] 0 [/ latex]. Диапазон синего графика — все действительные числа, [latex] \ mathbb {R} [/ latex].

    Пример 2:
    Определите область и диапазон каждого графика, изображенного ниже:

    График домена и диапазона: Синий график — это тригонометрическая функция [latex] f (x) = sin (x) [/ latex] с доменом [latex] \ mathbb {R} [/ latex] и ограниченным диапазон [латекс] -1 \ leq y \ leq 1 [/ latex] (выходные значения существуют только в диапазоне от [latex] -1 [/ latex] до [latex] 1 [/ latex].Красный график — это функция [latex] f (x) = — \ sqrt {x} [/ latex] с ограниченным доменом [latex] x \ geq 0 [/ latex], а также с ограниченным диапазоном [latex] у \ leq0 [/ латекс].

    Области рациональных и радикальных функций

    Рациональные и радикальные выражения имеют ограничения на их области, которые можно найти алгебраически или графически.

    Цели обучения

    Вычислить область определения рациональной или радикальной функции, найдя значения, для которых она не определена

    Ключевые выводы

    Ключевые моменты
    • Рациональное выражение — это отношение двух многочленов.Это может быть выражено как [латекс] \ displaystyle \ frac {P (x)} {Q (x)} [/ latex].
    • Область рационального выражения установлена ​​так, что знаменатель не может быть равен нулю. Следовательно, учитывая [латекс] \ displaystyle \ frac {P (x)} {Q (x)} [/ latex], [latex] Q (x) \ neq 0 [/ latex].
    • Чтобы определить область рационального выражения, установите знаменатель равным нулю, а затем решите относительно [латекс] x [/ латекс]. Все значения [latex] x [/ latex], кроме тех, которые удовлетворяют [latex] Q (x) = 0 [/ latex], являются областью выражения.
    • Радикальное выражение выражается как [latex] \ sqrt x [/ latex] и может иметь другие корни, кроме квадратного.
    • Радикальная функция выражается как [latex] f (x) = \ sqrt x [/ latex] (обычно называемая просто «функцией квадратного корня») — это функция, которая отображает набор неотрицательных действительных чисел на сам.
    • Чтобы определить домен радикального выражения, установите подкоренное выражение равным нулю, а затем найдите [латекс] x [/ latex]. Все значения [latex] x [/ latex], кроме тех, которые удовлетворяют [latex] \ sqrt x = 0 [/ latex], являются областью выражения.
    Ключевые термины
    • radicand : число или выражение под знаком корня.
    • рациональное выражение : выражение, которое можно записать как частное двух многочленов.

    Поиск областей рациональных функций

    Рациональное выражение — это выражение, которое можно записать как отношение двух полиномиальных функций. Несмотря на то, что это называется рациональным выражением, ни коэффициенты многочленов, ни значения, принимаемые функцией, не обязательно являются рациональными числами.В случае одной переменной, [latex] x [/ latex], выражение называется рациональным тогда и только тогда, когда оно может быть записано в форме:

    [латекс] \ displaystyle \ frac {P (x)} {Q (x)} [/ латекс]

    где [latex] P (x) [/ latex] и [latex] Q (x) [/ latex] являются полиномиальными функциями от [latex] x [/ latex], а [latex] Q (x) [/ latex] равно не нулевой многочлен [латекс] (Q (x) \ neq 0) [/ latex].

    Область рационального выражения — это набор всех точек, знаменатель которых не равен нулю.2-5) = 0 [/ латекс]

    Для решения разделите обе стороны на [латекс] 2 [/ латекс], добавьте [латекс] 5 [/ латекс] к обеим сторонам, а затем извлеките квадратный корень из обеих сторон, чтобы получить:

    [латекс] \ displaystyle x = \ pm \ sqrt {5} [/ latex].

    Следовательно, домен представляет собой набор всех действительных чисел, кроме квадратного корня из пяти или отрицательного квадратного корня из пяти.

    Обратите внимание на график функции ниже. При значениях [латекс] x = \ pm \ sqrt {5} [/ latex] (что приблизительно равно [латекс] \ pm 2.2-2 \ справа)} {x} [/ латекс]

    Алгебраически область — это набор всех действительных чисел, кроме нуля, поскольку знаменатель не может быть равен нулю. Один из способов определить это — посмотреть на это графически. Мы можем видеть, что график не является непрерывным при [latex] x = 0 [/ latex], что указывает на то, что домен состоит из всех чисел, кроме [latex] x = 0 [/ latex]. Это имеет смысл, потому что при [latex] x = 0 [/ latex] нам пришлось бы разделить на ноль, который не определен. Линии графика становятся все ближе и ближе к значению [latex] x = 0 [/ latex], но никогда не соприкасаются.2-2 \ right)} {x} [/ латекс]. Чтобы определить область действия этой функции, мы можем построить график и найти, где функция не существует, в данном случае, когда [latex] x = 0 [/ latex].

    Поиск областей радикальных функций

    Функция главного квадратного корня [latex] f (x) = \ sqrt x [/ latex] (обычно называемая просто «функцией квадратного корня») — это функция, которая отображает набор неотрицательных действительных чисел на себя.

    Радикальная функция: Функция [latex] f (x) = \ sqrt x [/ latex] состоит из ограниченной области [latex] x \ geq 0 [/ latex] или неотрицательных действительных чисел, поскольку мы нельзя извлечь квадратный корень из отрицательного числа.2 [/ latex] будет [latex] \ sqrt y = \ pm x [/ latex]). При построении графика корней важно помнить, что отрицательные значения [latex] x [/ latex] не будут давать действительные числа. Это будет объяснено далее в разделе о мнимых числах.

    Чтобы определить область определения радикальной функции алгебраически, найдите значения [latex] x [/ latex], для которых подкоренное выражение неотрицательно (установите его равным [latex] \ geq 0 [/ latex]), а затем решите для [ латекс] х [/ латекс]. Подкоренное число — это число или выражение под знаком корня.Все значения [latex] x [/ latex], кроме тех, которые удовлетворяют [latex] \ sqrt x \ geq 0 [/ latex], являются доменом функции.

    Пример 3: Область определения радикальной функции:

    [латекс] \ displaystyle f (x) = \ sqrt {x-3} +4 [/ латекс]

    Установите подкоренное выражение больше или равное нулю и решите для [latex] x [/ latex], чтобы найти ограничения для домена:

    [латекс] \ displaystyle {x-3} \ geq 0 [/ латекс]

    Следовательно [латекс] х \ geq 3 [/ латекс]. Итак, все действительные числа, большие или равные [latex] 3 [/ latex], являются областью определения функции.

    Радикальная функция : График уравнения: [латекс] f (x) = \ sqrt {x-3} +4 [/ latex]. Функция имеет область значений всех действительных чисел, больших или равных [латекс] 3 [/ латекс], как показано на графике выше.

    1.3 — Функции

    1.3 — Функции

    Определения

    Отношение
    Правило, которое связывает значение в домене со значением в диапазоне.
    Функция
    Функция — это отношение (правило), которое присваивает каждому элементу в домене ровно один элемент в диапазоне.
    Домен
    Набор всех значений, которые могут быть введены в функцию. То есть набор всех значения, которые может принимать независимая переменная. Графически домен — это совокупность всех x-координаты.
    Диапазон
    Набор всех значений, которые выводятся, когда функция оценивается на всех входах. значения из домена. То есть набор всех значений зависимой переменной может предполагать. Графически диапазон — это набор всех y-координат.
    Независимая переменная
    Обычно независимой переменной является x. Однако независимой переменной является переменная, которая может принимать разные значения независимо от другой переменной. Большая часть того, что мы собираемся сделать в этом классе, будет включать только один независимый переменная, но помните, что возможно иметь более одной независимой переменной.
    Зависимая переменная
    Обычно зависимой переменной является y.Однако зависимой переменной является переменная который определяется на основе значения независимой (ых) переменной (ей). Если функция записывается как y = 3x + 2, тогда y зависит от x, но x не зависит от y (в виде написано). Итак, x не зависит от y, но y зависит от x.
    Подразумеваемый домен
    Набор всех действительных чисел, для которых определено выражение.

    Чем функция отличается от отношения?

    Вот несколько рекомендаций по определению, является ли отношение функцией или нет.

    1. Каждый элемент в домене (x) должен соответствовать элементу в диапазоне (y).
    2. Каждый элемент в домене (x) может быть сопоставлен только с одним элементом в диапазон (y).
    3. Различные элементы в домене могут переходить к одному и тому же элементу в диапазон. Координата Y может повторяться, но координата x не может.
    4. Некоторые значения в диапазоне не обязательно использовать

    Обозначение функций

    Обозначение функций используется для именования функций для удобства.Представьте себе, если каждая функция в мире должна была начинаться с y =. Довольно скоро ты будешь запутаться в том, какой y = вы говорили о. Так работают антецеденты в английском языке. Если ты просто скажи «это» красный, ты на самом деле понятия не имею, о чем «это» вы говорите. Вам нужен другой способ называть вещи. Введите обозначение функции.

    Определение функции

    • f (x) = 3x + 2
    • г (x, y) = x 2 + 3y

    В этом примере f является функцией x.То есть x — независимая переменная, а значение f зависит от того, что такое x. Кроме того, g является функцией как x, так и y.

    Обозначение f (x) не означает f, умноженное на x. Это означает, что «значение f оценивается при x «или» значении f at x «или просто» f of x »

    Оценка функций

    f (3) = 3 (3) + 2 = 9 + 2 = 11

    f (3) не означает f умножить на 3. Это означает «значение f, вычисленное, когда x составляет 3 дюйма.

    f (t) = 3 (t) + 2 = 3t + 2

    Все, что указано в скобках в левой части функция (в данном случае t) подставляется вместо значения независимого переменная справа.

    f (x + h) = 3 (x + h) + 2 = 3x + 3h + 2

    Каждое появление независимой переменной заменяется количеством в скобках. А Распространенная ошибка — взять величину и применить к ней линейные или аффинные преобразования.

    ф (х + в)

    • f (x + h) не равно f (x) + h = 3x + 2 + h
    • f (x + h) не равно f (x) + f (h) = 3x + 2 + 3h + 2 = 3x + 3h + 4
    • f (x + h) равно 3 (x + h) + 2 = 3x + 3h + 2

    f (3x)

    • f (3x) не равно 3 * f (x) = 3 (3x + 2) = 9x + 6
    • Это равно 3 (3x) + 2 = 9x + 2

    Вы также указываете, какую функцию хотите использовать, когда используете обозначение функций.Рассмотрим

    г (2,1) = 2 2 + 3 (1) = 4 + 3 = 7

    Поскольку порядок независимых переменных в исходном определении был x, а затем y, функция g вычисляется, когда x = 2 и y = 1.

    г (1,2) — это совсем другое дело. В этом случае x = 1 и y = 2.

    г (1,2) = 1 2 + 3 (2) = 1 + 6 = 7

    Ладно, это не совсем другое, но обнаруживается по-другому.

    Кусочные определения

    Иногда функции немного сложнее, чем простые функции, которые мы описали до сих пор.Если разные правила используются для разных значений независимой переменной, тогда мы можем использовать кусочное определение.

    Учитывайте показанную функцию и следующие оценки.

    f (2,1) = 2 (2,1) = 4,2

    Поскольку 2.1 находится в интервале [1,3), мы используем вторую часть определения, f (х) = 2х.

    f (-2) = 3 — (-2) 2 = 3-4 = -1

    Поскольку -2 находится в интервале (-∞, -1), мы используем первую часть определение, f (x) = 3-x 2

    f (3) = 5 — (3) = 5 — 3 = 2

    Так как 3 не входит во вторую штуку, но входит в третью пьеса на антракте [3, + ∞) мы используем третью часть определения, f (x) = 5 — x.

    f (0) не определено

    Поскольку 0 не попадает ни в один из доменов, функция там не определена.

    Когда все области объединены, область определения функции f является множеством всех действительных чисел, кроме [-1,1). Вы также можете записать его как (-∞, -1) U [1, + ∞). Символ U обозначает объединение двух наборов.

    Кусочные функции и калькулятор

    Вы можете использовать кусочные функции в графическом калькуляторе. Калькуляторы TI82 и TI83 получить ключ [Тест], нажав [2 nd ] [Math].Под этим ключом вы найдете разные операторы проверки (равно, не равно, больше, больше или равно, меньше и меньше или равно).

    Когда калькулятор оценивает тестовое выражение (x <1), он возвращает значение 1, если оператор истинно и значение 0, если утверждение ложно. Это действительно хорошо работает с умножением, потому что умножение на 1 не изменит выражение, а умножение на 0 сделает выражение 0.

    y = (1 — x 2 ) * (x <-1)

    Рассмотрим только первую часть кусочного определения сверху.2). Если x не меньше отрицательного, тогда калькулятор вернет 0. Это неправильное значение для возврата (оно должно быть неопределенным), но оно будет отображаться на графике ось x и не будет отображаться, потому что ось уже там. Итак, у него есть видимость на графике правильно.

    Полная кусочная функция может быть определена для калькулятора как:

    y = (1-x 2 ) * (x <-1) + 5x * (1≤x и x <3) + (5-3x) * (x≥3)

    Ключевое слово «and» можно найти в меню [Test] Logic.Аналогично символы ≤ и ≥ также можно найти в меню [Тест]. Если вы попытаетесь положить 1≤x <3, как мы правильно, калькулятор интерпретирует это как 1≤3 и всегда возвращает true.

    Обязательно используйте какой-либо десятичный режим или режим точки при просмотре кусочного функция. В противном случае вы можете получить странные результаты.

    В поисках домена

    Подразумеваемый домен

    Мы определили подразумеваемый домен ранее. Это набор всех действительных чисел, в котором выражение определенный.

    1. Начните со всех действительных чисел
    2. Исключить любые значения независимой переменной, вызывающие деление на ноль
    3. Исключить любые значения независимой переменной, которые приводят к извлечению квадратного корня из отрицательное число. Квадратный корень можно заменить любым четным (4 , 6 и т. Д.) Корнем.

    Вам не , а не необходимо указывать ограничения, вытекающие из подразумеваемого домена. Другими словами, если в знаменателе стоит (x-2), вам не нужно указывать, что x не может быть 2.

    Прочие исключения

    Иногда вам нужно исключить другие значения.

    1. Проблемы с ограничениями — они используются в приложениях. вот несколько примеров
      • Если x — длина стороны треугольника, тогда x не может быть отрицательным
      • Если x — положение на качелях высотой 12 футов, в котором точка опоры должна быть позиционированным, то x должен быть в пределах от 0 до 12.
      • Если x — количество человек в комнате, то x не может быть отрицательным.
    2. Заявленные ограничения — иногда проблема просто выходит и говорит, что вы не можете использовать определенное значение, или что вы можете использовать только определенные значения. Это не подразумевается, так что их нужно заявить.

    Если вы когда-нибудь упростили функцию, и значение, которое было в подразумеваемой области больше не находится в подразумеваемой области, тогда это должно стать установленным ограничением. Например, разделите (x 2 -4) на (x-2). Подразумеваемая область значений x не может быть равна 2, потому что это приведет к делению на ноль.Однако, если вы разложите числитель на множители как (x-2) (x + 2), то (x-2) в числителе делится на (x-2) в знаменатель, и у вас останется только (x + 2). Тот факт, что x не может быть 2 больше не подразумевается простым x + 2 в числителе, поэтому теперь вы должны указать что x не может быть 2.

    Объединение доменов

    Когда у вас есть функция, которая представляет собой композицию из нескольких частей, значения в домене должны быть в состоянии использоваться во всех частях функции.Скажем, в числителе есть квадратный корень из x в нем (так что x должен быть неотрицательным), а в знаменателе есть x-2 (так что x не может быть 2). Когда вы объединяете эти домены, вы получаете все неотрицательные значения x, кроме 2.

    Другими словами, область действия функции — это пересечение всех областей индивидуальной части.

    Теперь предположим, что в числителе есть квадратный корень из x (так что x должен быть неотрицательным) и в знаменателе стоит x + 2 (так что x не может быть -2).Когда вы объединяете эти домены, вы получаете все неотрицательные значения x, кроме -2. Ну, вам не нужно утверждать, что x не может быть -2 поскольку вы уже сказали, что x неотрицательно. Итак, в этом случае доменом будут все неотрицательные значения x.

    функций: домен и диапазон | Purplemath

    Purplemath

    Вернемся к теме доменов и диапазонов.

    Когда функции вводятся впервые, вы, вероятно, будете иметь дело с некоторыми упрощенными «функциями» и отношениями, обычно представляющими собой просто наборы точек. Это не будут очень полезными или интересными функциями и связями, но ваш текст хочет, чтобы вы получили представление о том, что такое домен и диапазон функции. Небольшие наборы точек, как правило, являются простейшими видами отношений, поэтому ваша книга начинается с них.

    Например:

    MathHelp.com

    • Укажите область и диапазон следующего отношения. Отношение — это функция?

    {(2, –3), (4, 6), (3, –1), (6, 6), (2, 3)}

    Приведенный выше список точек, являющийся отношением между определенными x и некоторыми y , является отношением.Домен — это все значения x , а диапазон — все значения y . Чтобы указать домен и диапазон, я просто перечисляю значения без дублирования:

    домен: {2, 3, 4, 6}

    диапазон: {–3, –1, 3, 6}

    (Обычно эти значения перечисляются в числовом порядке, но не требуется . Наборы называются «неупорядоченными списками», поэтому вы можете перечислять числа в любом порядке.Только не дублируйте: технически повторения в подходах допустимы, но большинство инструкторов рассчитывают на это.)

    Хотя данный набор действительно представляет отношение (поскольку x и y связаны друг с другом), набор, который они мне дали, содержит две точки с одинаковым значением x : (2, –3) и (2, 3). Поскольку x = 2 дает мне два возможных назначения (то есть два возможных значения y ), то это отношение не является функцией.

    Обратите внимание, что все, что мне нужно было сделать, чтобы проверить, является ли отношение функцией, — это найти повторяющиеся значения x . Если вы обнаружите какие-либо повторяющиеся значения x , то разные значения y означают, что у вас нет функции. Помните: чтобы отношение было функцией, каждое значение x должно перейти к одному, и только одному значению , y .

    • Укажите область и диапазон следующего отношения.Отношение — это функция?

    {(–3, 5), (–2, 5), (–1, 5), (0, 5), (1, 5), (2, 5)}

    Я просто перечислю значения x для домена и значения y для диапазона:

    .

    домен: {–3, –2, –1, 0, 1, 2}

    диапазон: {5}

    Это еще один пример «скучной» функции, такой же, как в примере на предыдущей странице: каждое последнее значение x соответствует одному и тому же значению y .Но каждое значение x отличается, поэтому, пока скучно,

    это отношение действительно является функцией.

    Фактически, эти точки лежат на горизонтальной прямой y = 5.


    Между прочим, имя набора, в котором есть только один элемент, как и набор «диапазон» выше, — «singleton». Таким образом, диапазон также может быть указан как «синглтон из 5»


    Есть еще один случай нахождения домена и диапазона функций.Они дадут вам функцию и попросят найти домен (и, возможно, диапазон). На этом этапе вашей математической карьеры я видел только (или даже могу придумать) две вещи, которые вам нужно будет проверить, чтобы определить область определения функции, которую они вам дадут, и эти две вещи являются знаменателями и квадратные корни.

    • Определите область и диапазон данной функции:

    Домен — это все значения, которые разрешено принимать x .Единственная проблема, с которой я сталкиваюсь с этой функцией, заключается в том, что мне нужно быть осторожным, чтобы не делить на ноль. Таким образом, единственные значения, которые x не могут принять, — это те, которые вызывают деление на ноль. Поэтому я установлю знаменатель равным нулю и решу; в моем домене будет все остальное.

    x 2 x — 2 = 0

    ( x — 2) ( x + 1) = 0

    x = 2 или x = –1

    Тогда домен будет «все x не равны –1 или 2».

    Диапазон немного сложнее, поэтому они могут не просить об этом. В общем, они хотят, чтобы вы построили график функции и нашли диапазон по картинке. В этом случае:

    Как видно из моего рисунка, график «покрывает» все значения и ; то есть график будет идти настолько низко, насколько мне нравится, и также подниматься настолько высоко, насколько мне нравится. Для любой точки оси y , независимо от того, насколько высоко вверх или вниз, я могу перейти от этой точки вправо или влево и, в конце концов, пересечу график.Поскольку в конечном итоге график будет охватывать все возможные значения y , тогда:

    диапазон — «все действительные числа».

    • Определите область и диапазон данной функции:

    Домен — это все значения, которые могут принимать x .Единственная проблема, с которой я столкнулся с этой функцией, заключается в том, что у меня не может быть отрицательного числа внутри квадратного корня. Итак, я установлю внутренности больше или равными нулю и решу. Результатом будет мой домен:

    –2 x + 3 ≥ 0

    –2 x ≥ –3

    2 x ≤ 3

    x ≤ 3/2 = 1,5

    Тогда домен будет «все x ≤ 3/2».

    Для диапазона требуется график. Мне нужно быть осторожным при построении графика радикалов:

    График начинается с y = 0 и идет вниз (направление влево) оттуда. Хотя график идет вниз очень медленно, я знаю, что в конечном итоге я смогу опуститься настолько низко, насколько захочу (выбрав достаточно большой размер x ). Кроме того, по моему опыту работы с графиками я знаю, что график никогда не начнет восстанавливаться. Тогда:

    диапазон: «все y ≤ 0».

    Партнер

    • Определите область и диапазон данной функции:

    y = — x 4 + 4

    Это просто многочлен садового разнообразия.Здесь нет знаменателей (поэтому нет проблем с делением на ноль) и нет радикалов (поэтому нет проблем с извлечением квадратного корня из отрицательного числа). С полиномом проблем нет. Нет значений, которые я не могу подключить для x . Когда у меня есть полином, ответ для домена всегда :

    Диапазон будет варьироваться от полинома к полиному, и они, вероятно, даже не спросят, но когда они это сделают, я смотрю на картинку:

    График идет только до y = 4, но он будет идти настолько низко, насколько мне нравится.Тогда:

    Диапазон: «все y ≤ 4».


    URL: https://www.purplemath.com/modules/fcns2.htm

    Домен и диапазон

    Домен и диапазон функции — это все возможные значения независимой переменной x, для которой определено y.Диапазон функции — это все возможные значения зависимой переменной y.

    В приведенном ниже примере показаны два различных способа представления функции: в виде таблицы функций и в виде набора координат.

    или {(2, 4), (3, 8), (5,2), (6,9), (8,3)}

    Несмотря на то, что они представлены по-разному, это одна и та же функция, а область определения функции — x = {2, 3, 5, 6, 8}, а диапазон — y = {4, 8, 2, 9, 3}.Вот как вы можете определить домен и диапазон для дискретных функций. Порядок, в котором вы указываете значения, не имеет значения. Но как определить домен и диапазон для недискретных функций?

    Пример:

    f (x) = x 2

    Функция f (x) = x 2 имеет область значений всех действительных чисел (x может быть любым) и диапазон, который больше или равен нулю.

    Два способа записи домена и диапазона функции: интервальная нотация и заданная нотация.

    Интервальное обозначение

    При использовании обозначения интервалов домен и диапазон записываются как интервалы значений. Для f (x) = x 2 домен в интервальной записи:

    D: (-∞, ∞)

    D означает, что вы говорите о домене, а (-∞, ∞), читаемое как от отрицательной бесконечности до положительной бесконечности, — это еще один способ сказать, что домен — это «все действительные числа».

    Диапазон f (x) = x 2 в интервальной записи:

    R: [0, ∞)

    R означает, что вы говорите о диапазоне.Обратите внимание, что вместо скобок для 0 используется скобка. Это связано с тем, что диапазон функции включает 0 при x = 0. Диапазон функции исключает ∞ (это делает каждая функция), поэтому мы используем круглую скобку.

    На графике вы знаете, когда функция включает или исключает конечную точку, потому что конечная точка будет открытой или закрытой.

    Установить обозначение

    При использовании обозначения набора мы используем символы неравенства для описания области и диапазона как набора значений. Домены и диапазоны, использованные в примерах дискретных функций, были упрощенными версиями обозначений множеств.В обозначениях множеств используется много разных символов, но здесь будут представлены только самые основные структуры.

    Область значений f (x) = x 2 в заданных обозначениях:

    D: {x | x∈ℝ}

    Опять же, D указывает на домен. Знак «|» означает «такое, что» символ ∈ означает «элемент из», а «ℝ» означает «все действительные числа».

    Собирая все вместе, это утверждение может быть прочитано как «домен — это набор всех x таких, что x является элементом всех действительных чисел».

    Диапазон f (x) = x 2 в заданных обозначениях:

    R: {y | y ≥ 0}

    R указывает дальность действия.При использовании обозначения набора символы неравенства, такие как ≥, используются для описания области и диапазона. Следовательно, это утверждение может быть прочитано как «диапазон — это набор всех y таких, что y больше или равно нулю».

    Добавить комментарий

    Ваш адрес email не будет опубликован. Обязательные поля помечены *